Rew

April 11, 2018 | Author: Margenete Casiano | Category: Audit, Internal Control, Debits And Credits, Bad Debt, Deferred Tax
Share Embed Donate


Short Description

quizzer...

Description



A1 PASSERS TRAINING, RESEARCH, REVIEW & DEVELOPMENT COMPANY 2nd Floor Sommerset Bldg., Lopez Jaena St. Jaro, Iloilo City Tel. No.: (033) 320-2728; 09106547262 Email Address: [email protected] BOARD OF CERTIFIED PUBLIC ACCOUNTANT CERTIFIED PUBLIC ACCOUNTANT Licensure Examination SET A

THEORY OF ACCOUNT GENERAL INSTRUCTIONS: 1. This test booklet contains 100 test questions. 2. Read INSTRUCTIONS TO EXAMINEES printed on your answer sheet. 3. Shade only one (1) box for each question on your answer sheets. Two or more boxes shaded will invalidate your answer. 4. AVOID ERASURES. INSTRUCTIONS: 1. Detach one (1) answer sheet from the bottom of your Examinee ID/Answer Sheet Set. 2. Write the subject title “THEORY OF ACCOUNT” on the box provided. 3. Shade Set Box “A” on your answer sheet if your test booklet is Set A; Set Box “B” if your test booklet is Set B. __________________________________________________________________________________________

1. General –purpose financial statements are the product of a. Financial accounting b. Managerial accounting c. Both financial and managerial accounting d. Neither financial accounting nor managerial accounting 2. Which of the following statements is not an objective of financial reporting? a. To provide information that is useful in investment and credit decision. b. To provide information about resources claims against those resources and changes in them. c. To provide information on the liquidation value of an entity. d. To provide information that is useful in assessing cash flow prospects. 3. The purpose of the International Accounting Standards Board is to a. Issue enforceable standard which regulate the financial reporting of multinationals. b. Develop a uniform currency in which the financial transactions of entities throughout the world would be measured. c. Develop a single set of high quality IFRS. d. Arbitrate accounting disputes between auditors and international entities. 4. The underlying theme of the conceptual framework is a. Decision usefulness c. Reliability b. Understand ability d. Comparability 5. The Conceptual Framework includes all of the following except a. Objective of financial reporting d. Qualitative characteristics of accounting b. Supplementary information information c. Elements of financial statements 6. Which of the following is a fundamental quality of useful accounting information? a. Conservatism c. Faithful representation b. Comparability d. Consistency 7. In the Conceptual Framework an enhancing qualitative characteristic is a. Predictive value c. Timeliness b. free from error d. Confirmatory value 8. Which of the following is not a basic element of financial statements? a. Asset c. Equity b. Statement of financial position d. Income 9. Which of the following is not a basic assumption underlying the financial accounting structure? a. Economic entity assumption c. Periodicity assumption b. Going concern assumption d. Historical cost assumption 10. During the lifetime of an entity accountants produce statements at artificial points in time. a. Objectivity c. Economics entity b. Periodicity d. Going concern A1 PASSERS TRAINING, RESEARCH, REVIEW AND DEVELOPMENT COMPANY// 1

11. When revenue is generally recognized? a. When cash is received c. When production is completed b. When the warranty expire d. When the sale occurs 12. Which of the following is not a required component of financial statements? a. President’s letter to shareholders c. Income statement b. Statement of financial position d. Notes to financial statements 13. Under Philippine Financial Reporting Standards note to the financial statements a. Must be quantifiable. b. Must qualify as an element. c. Amplify or explain items presented in the main body of the financial statements. d. All of the choice are correct regarding notes to the financial statements. 14. Nominal accounts are also called a. Temporary accounts c. Real accounts b. Permanent accounts d. Mixed account 15. A trial balance may prove that debits and credit are equal but a. An amount could be entered in the wrong account. b. A transaction could have been entered twice c. A Transaction could have been omitted. d. All of these. 16. A journal entry to record a receipt of rent revenue in advance will include a a. Debit to revenue c. Credit to cash b. Credit to rent revenue d. Credit to unearned rent 17. An Adjusting entry to record an receipt and accrued expense involves a debit to a. Expense account and a credit to an prepaid account. b. Expense account and a credit to cash. c. Expense account and a credit to a liability account. d. Liability account and a credit to an expense account. 18. Reversing entries do not apply to which of the following items? a. Unearned revenue c. Prepaid insurance b. Accrued wages d. Depreciation 19. The statement of financial position information is useful for all of the following except a. To compute rates of return c. To evaluate capital structure b. to analyze cash inflows and outflow for d. To assess future cash flow the period 20. A segment is to be reported separately when the segment revenue exceeds 10% of a. Total combined revenue of all segments reporting profits. b. Total revenue of all the entity‘s industry segments. c. Total export and foreign sales. d. Combined net income of all segment reporting profits. 21. All of the following statements are true regarding interim reporting, except a. PFRS requires a complete set of financial statements at the interim reporting date. b. PFRS requires entities to expense interim amount like advertising expenditures that could Benefit later interim period. c. PFRS allows a condensed set or complete set of financial statements. d. No accruals or deferrals in anticipation of future events during the year should be reported. 22. Which of the following would represent the least likely use of an income statement a. Use by customers to determine an entity’s ability to provide needed goods and services. b. Use by labor union s to examine earnings closely as a basis for salary discussions. c. Use by government agencies to formulate tax and economic policy. d. Use by investors interested in the financial position of the entity. 23. A change in accounting policy requires what kind of adjustment to the financial statements? a. Current period adjustment c. Retrospective adjustment b. Prospective adjustment d. Current and prospective adjustment 24. Changes in accounting estimate affect report amounts a. Retrospectively only c. Currently and prospectively b. Prospectively only d. Currently and retrospectively 25. Which of the following does not appear in a statement of retained earnings? a. Net loss c. Preference share dividends b. Prior period adjustment d. Other comprehensive income 26. Which is not acceptable in displaying the components of other comprehensive income? a. Combined statement of retained earning c. Combined statement of comprehensive b. Second income statement income d. All of the above are acceptable A1 PASSERS TRAINING, RESEARCH, REVIEW AND DEVELOPMENT COMPANY// 2

27. Which of the following items should not be included in “cash”? a. Coins and currency in the cash register b. Checks from other parties presently in the cash register c. Amounts on deposit in checking account at the bank d. Postage stamps on hand 28. A cash equivalent is a short-term highly liquid investment that is readily convertible into know amount of cash and a. Is acceptable as a means to pay current liabilities. b. Has a current market value that is greater than original cost c. Bears an interest rate that at least equal to the prime rate of interest. d. Is so near its maturity than it presents insignificant risk of changes in interest rates. 29. All of the following are problems associated with the valuation of accounts receivable, except a. Uncollectible accounts c. Cash discounts under the net method b. Returns d. Allowances granted 30. Which of the following methods of determining bad debt expense does not properly match expense and revenue? a. Charging bad debts with a percentage of sales under the allowance method. b. Charging bad debts with an amount derived from percentage of accounts receivable under the allowance method. c. Charging bad debts with an amount derived from aging accounts receivable under the allowance method. d. Charging bad debts as accounts are written off as uncollectible. 31. Which of the following statements best describes the term “liabilities”? a. An excess of equity over current assets b. Resources to meet financial commitment as they fall due c. The residual interest in the assets of the entity after deducting all of its liabilities d. A present obligation of the entity arising fro past events 32. Which of the following statements best describes the term ‘financial position’? a. The net income and expenses of an entity b. The financial assets less financial liabilities of an entity c. The potential to contribute to the flow of cash and cash equivalents to the entity d. The assets liabilities and equity of an entity 33. Which of the following should not be taken into account when determining the cost of inventories? a. Storage cost of part-finished goods b. Trade discounts Recoverable purchase taxes c. Recoverable purchase taxes d. Import duties on shipping of inventory inward 34. Which of the following costs should be included in inventory valuation? a. Administrative costs c. Storage costs relating to finish goods b. Abnormal material usage d. Fixed production overheads 35. Which classification of the cash flow arising from the proceeds from an earthquake disaster settlement would be most appropriate? a. Cash flows from operating activities d. Does not appear in the statement of cash b. Cash flows from investing activities flows c. Cash flows from financing activities 36. Which of the following should be treated as a change in accounting policy? I. A new accounting policy of capitalizing development costs as a project has become eligible for capitalization for the first time. II. A new policy resulting from the requirements of a new PFRS. III. To provide more relevant information items of property, plant and equipment are now being measured at fair value, whereas they had previously been measured at cost. IV. An entity engaging in construction contract for the first time needs an accounting policy to deal with this. a. I, II, III and IV c. II and III only b. I, and II only d. I and IV only 37. Which of the following statements in relation to deferred tax is true? I. Deferred tax liabilities are the amounts of income taxes payable in future periods in respect of taxable temporary differences. II. Deferred tax assets are the amounts of income taxes recoverable in future periods in respect of deductible permanent differences. a. I only c. Both I and II b. II only d. Neither I nor II A1 PASSERS TRAINING, RESEARCH, REVIEW AND DEVELOPMENT COMPANY// 3

38. Which term best describes the removal of an asset from the statement of financial position? a. De recognition c. Write off b. Impairment d. Depreciation 39. Which of the following statements is correct? a. Assets are depreciated even if their fair value exceeds carrying amount b. Land and buildings are not accounted separately when acquired together c. A noncurrent asset acquired as the result of an exchange of assets is not recognized d. A gain .on disposal of a noncurrent asset is classified as revenue 40. Which should not be capitalized a cost of properly plant and equipment? a. Cost of excess materials resulting from a purchasing error b. Cost of testing whether the asset works correctly c. Initial delivery and handling cost d. Cost of preparing the site for installation 41. Which should be included in the cost of an item of properly plant and equipment? a. Initial operating losses while demand c. Costs of training staff on the new asset builds up d. Installation and assembly costs b. Apportioned general overhead cost 42. Which of the following conditions does not apply to the recognition of revenue for transactions involving the rendering of services? a. The amount of revenue can be measured reliably b. It is probable that payment for the services shall be received by the relinquished c. Ownership has been transferred to the buyer d. Significant risks and rewards of ownership have been transferred from the seller to the buyer 43. Which must not be satisfied before revenue from the sale of goods should be recognized? a. Revenue can be reliably measured b. Managerial control over the goods sold has been relinquished c. Ownership has been transferred to the buyer d. Significant risks and rewards of ownership has been transferred from the seller to the buyer 44. Which of the following statements best describes other lone-term employee benefits”? a. Benefits not falling due wholly within twelve months of the end of the period in which the service is service is rendered b. Benefits which fall due within twelve months often end of the period in which the service is rendered c. Benefits payable as a result of an entity’s decision to end an employee’s employment before the normal retirement date d. Benefits which are payable after completion of employment 45. Which of the following statements is incorrect in relation to government grant? a. Any adjustment needed when a government grant becomes repayable is accounted for as a change in accounting estimate. b. In respect of loans from the government at an interests rate 0% an imputed interest charge is required. c. Where condition apply to a government grant it should only be recognized when there is reasonable assurance that the conditions will be met. d. A government grant that becomes receivable as compensation for losses already incurred should be recognized as income of the period in which it becomes receivable. 46. Which statement about the capitalization of borrowing costs is true? a. If Funds come from general borrowings the amount to the capitalized is based on the weighted average cost of borrowing. b. Capitalization always continues until the asset is brought into use. c. Capitalization always commences as soon as expenditure f the asset is incurred. d. Capitalization always commences as soon as interest on relevant borrowing is being incurred. 47. Which of the following treatments is required for borrowing costs incurred that are directly attributable to the construction of a qualifying asset? I.Recognize as an expense in the period incurred. II.Capitalize as a part of the cost of the asset. a. I only c. Either I or II b. II only d. Neither I nor II 48. An entity completely the following transactions in the current year: I.Sold a car to the uncle of the entity’s finance director. II.Sold goods to another entity owned by the daughter of the entity’s managing director. Which transaction would require disclosure in the financial statements of the entity? a. Neither I nor II c. II only b. I only d. Both I and II A1 PASSERS TRAINING, RESEARCH, REVIEW AND DEVELOPMENT COMPANY// 4

49. Which of the following is not a related party of an entity? a. A shareholder of the entity owning 30% of the ordinary share capital b. An entity providing banking facilities to the entity c. An associate of the entity d. Key management personnel of the entity 50. Which of the following statements about dividends is true? I.Dividends in respect of ordinary shares are debited directly in equity. II.Dividends in respect of redeemable preferences shares are debited directly in equity. a. I only c. Both I and II b. II only d. Neither I nor II 51. Which of the following statements with respect to interim reporting is true? I.It is necessary to count inventories in full at the end of each interim accounting period. II.The net realizable value is determined by reference to selling prices at the interim date. a. I only c. Both I and II b. II only d. Neither I nor II 52. Which of the following terms best describes the higher of fair value less cost of disposal and value in use? a. Recoverable amount c. Depreciable amount b. Revalued amount d. Carrying amount 53. Which of the following statements in relation to a contingent liability is true? I. An obligation as a result of the entity creating a valid expectation that it will discharge its responsibilities is a contingent liability. II. A present obligation that arises from past events but cannot be reliably measured is a contingent liability. a. I only c. Both I and II b. II only d. Neither I nor II 54. A provision should be recognized for which of the following? a. Future operating losses b. Obligations under insurance contracts c. Reductions in fair value of financial instruments d. Obligations for plant decommissioning costs 55. Which of the following is not relevant in determining the useful life of an intangible asset? a. Obsolescence c. The expected usage of the asset b. Expected action of competitor d. The residual value of the asset 56. A brand name that was acquired separately should initially be recognized at a. Recoverable amount c. Fair value b. Either cost or fair value at the choice of d. Cost the acquirer 57. Which of the following statements is true? I. Intangible assets cannot be treated as having an indefinite useful life. II. Intangible assets with a finite useful life should be measured at cost and tested annually for impairment. a. I only c. Both I and II b. II only d. Neither I nor II 58. Which of the following additional disclosures must be made when an entity chooses the cost model as its accounting policy for investment properly? a. The fair value of the property c. The value in use of the property b. The present value of the property d. The net realizable value of the property 59. Which of the following disclosures should be made when the fair value model has been adopted for investment property? a. Depreciation method used c. Useful life or depreciation rate used b. The amount of impairment loss d. Net gains or losses from fair value recognized adjustments 60. Where should changes in the fair value of a herd of cattle be recognized in the financial statements? a. In profit or loss c .In profit or loss or other comprehensive b. In other comprehensive income income d. In the statement of cash flows 61. Which of the following reports is not a component of the financial statement? a. Statement of financial position c. Director’s report b. Statement of changes in equity d. Notes to the financial statements

A1 PASSERS TRAINING, RESEARCH, REVIEW AND DEVELOPMENT COMPANY// 5

62. Which of the following information is not specifically a required disclosure? a. Name of the reporting entity or other means of identification, and any change in that information from the previous year. b. Names of major shareholders of the entity. c. Level of rounding used in presenting the financial statements. d. Whether the financial statements cover the individual entity or a ground of entities. 63. Which one of the following is not required to be presented as minimum information on the face of the statement of financial position? a. Investment property c. Biological assets b. Investments accounted under the equity d. Contingent liability method 64. Inventories are defined by all of the following except a. Used in the production or supply of goods and services for administrative purposes. b. Held for sale in the ordinary course of business. c. In the process of production for such sale. d. In the form of materials or supplies to be consumed in the production process or the rendering of services. 65. How should an entity disclose the dividends received in the statement of cash flows? a. Operating cash inflow b. Either as operating cash inflow or as financing cash inflow c. Either as operating cash inflow or as financing cash inflow d. As an adjustment in the “operating activities” section 66. At the end of the current reporting period an entity carried a receivable from a major customer. The customer declared bankruptcy after the end of reporting period but prior to authorization of financial statements. How should the entity account for this event? a. Disclose in the notes the fact that the customer declared bankruptcy. b. Make a provision for this post-reporting period event c. Ignore the event d. Reverse the sale pertaining to the receivable and treat it as an error. 67. The initial operating losses should be a. Deferred and authorized over a reasonable period of time. b. Expensed and charged to the income statement. c. Capitalized as part of the cost of plant as a directly attributable cost. d. Charged to retain earning. 68. An entity owns a fleet of cars and ships. The entity decided to revalue its property, plant and equipment. Which option should be selected in relation to the revaluation? a. Revalue only one-half of each class of property, plant and equipment. b. Revalue an entire class of property, plant and equipment. c. Revalue one ship at a time as it is easier than revaluing all ships altogether. d. Since assets are being revalue regularly, here is no need to depreciate. 69. The classification of a lease as either an operating or finance lease is based on a. The length of the lease. b. The transfer of the risks and rewards of ownership. c. The minimum lease payments being at least 50% of the fair value. d. The economic life of the asset. 70. The accounting concept that is principally used to classify leases into operating and finance is a. Substance over form c. Neutrality b. Prudence d. Completeness 71. Which situation would prima facie lead to a lease being classified as an operating lease? a. Transfer of ownership to the lessee at the end of the lease term. b. Option to purchase at a value below the fair value of the asset. c. The lease term is for a major part of the asset’s life. d. The present value of the minimum lease payments is 50% of the fair value of the asset. 72. Revenue from an artistic performance is recognized once a. The audience register for the event online c. Cash has been received from the ticket b. The tickets for the concert are sold. sales. d. The event takes place. 73. In the case of a nonmonetary grant, which of the following is prescribed? a. Record the asset at replacement cost and the grant at a nominal value. b. Record the grant at a value estimate by management. c. Record both the grant and the asset at fair value of the nonmonetary asset. d. Record only the asset at fair value and not recognize the fair value of the grant. A1 PASSERS TRAINING, RESEARCH, REVIEW AND DEVELOPMENT COMPANY// 6

74. Which of the following should not be considered a “qualifying asset” in relation to capitalization of borrowing cost? a. A power generation plant that normally takes two years to construct. b. An expensive private jet that can be purchased from a local vendor. c. A toll bridge that usually takes more than a year to build. d. A ship that normally takes one to two years to complete. 75. The equity method is not required to be applied when the associate has been acquired and held with a view to its disposal within what time period? a. Six months c. Two years b. Twelve months d. In the near future 76. How is goodwill arising on the acquisition of an associate dealt with in the financial statements? a. It is amortized. b. It is impairment tested individually. c. It is written off against profit or loss. d. Good it’s not recognized separately within the carrying amount of the investment. 77. Which of the following assets or liabilities should be considered nonmonetary? a. Trade receivables c. Accrued expense and other payables b. Deferred tax liabilities d. Taxes payable 78. Which of the following assets is not a financial asset? a. Cash b. An equity instrument of other entity c. A contract that may or will be settled in the entity’s own instrument and is not classified as an equity instrument of the period. d. Prepaid expense 79. Earnings per share are calculated before accounting for which items? a. Preference dividend for the period c. Taxation b. Ordinary dividend d. Minority 80. If a bonus issue occurs between the year-end and the date financial statements are authorized a. EPS both for the current and the previous year are adjusted. b. EPS for the current year only is adjusted. c. No adjustment is made to EPS. d. Dilute EPS only is adjusted. 81. Publicly traded entities are encourage to provide interim financial reports a. At least at the half year and within 60 days of the end of interim period. b. Within a month of the half year-end. c. On a quarterly basis. d. Whenever the entity wishes. 82. If an entity does not prepare interim financial reports a. The year-end financial statements are deemed not to comply with FRRS. b. The year-end financial statements compliance with PFRS is not affected. c. The year-end financial statements will not be acceptable under local legislation. d. Interim financial reports should be included in the year-end financial statements. 83. Value in use is a. The market value. b. The discounted present value of future cash flows arising from use the asset and from its disposal. c. The higher of fair value the asset is recognized in the statement of financial position. d. The amount at which the asset is recognized in the statement of financial position. 84. Estimate of future cash flows in determining value in use normally would cover projections over maximum of a. Five years c. Fifteen years b. Ten years d. Twenty years 85. Which of the following cash flows should not be included in calculating value in use? a. Cash flows from disposal. b. Income tax payment. c. Cash flows from the sale of assets produced by the asset. d. Cash outflows incurred to generate the cash inflows the continuing use of the asset. 86. A cash-generating unit is a. The smallest business segment. b. Any grouping of assets that generates cash flows. c. Any group of assets that are reported separately to management. d. The smallest group of assets that generates independent cash flows from continuing use. 87. When allocating impairment loss such a loss should reduce the carrying amount of which asset first? a. Property, plant, and equipment c. Goodwill b. Intangible assets d. Current assets

A1 PASSERS TRAINING, RESEARCH, REVIEW AND DEVELOPMENT COMPANY// 7

88. An investment property should be measured initially at a. Cost b. Cost less accumulated impairment losses. c. Depreciable cost less accumulated impairment losses. d. Fair value less accumulated impairment losses. 89. A gain arising from change in fair value of an investment property for which an entity has opted to use the fair value model is recognized in a. Net profit for the year c. Valuation reverses in the shareholders b. General reverses in the shareholders equity equity d. Retained earning 90. Generally speaking biological assets relating to agricultural activity should be measured using a. Historical cost c. A fair value approach b .Historical cost less depreciation less d. Net realizable value impairment 91. Where there is a production cycle of more than one year for a biological asset, separate disclosure is encouraged for a. Physical change only c. Total change in value b. Price change only d. Physical change and price change 92. How should the income from discontinued operation be presented in the income statement? a. The entity should disclose a single amount on the face of the income statement below the income from continuing operation. b. The amounts from discontinued operation should be broken down over each category of revenue and expense. c. Discontinued operation should be shown as a movement on retained earning. d. discontinued operation should be shown as a line item after gross profit. 93. Which of the following criteria does not have be met in order for an operation to be classified as discontinued? a. The operation should represent a separate major line of business or geographical area. b. The operation is part of a single plan to dispose of a separate major line of business or geographical. c. The operation is a subsidiary acquired exclusively with a view to resale. d. The operation must be sold within three months of the year-end. 94. How should the assets and liabilities of a disposal group classified as held for sale be shown in the statement of financial position? a. The assets and liabilities should be offset and presented as a single amount. b. The assets of the disposal group should be shown separately from other assets and the liabilities should be shown separately from other liabilities. c. The assets and liabilities of the disposal group should not be presented. d. There should be no separate disclosure of assets and liabilities that form part of a disposal group. 95. Which is not a characteristic of the “full cost” method of accounting in the oil and gas industry? a. All costs incurred in acquiring exploring and developing within a defined cost center are capitalization and amortized. b. Costs are capitalized even if a specific project in a cost center was a failure. c. Costs of unsuccessful acquisition and exploration activities are charged to expense. d. Exploration and evaluation asset is classified either as tangible asset or an intangible asset according to the nature of the asset. 96. Which expenditures would never qualify as an exploration and evaluation asset? a. Expenditures for acquisition of right to explore. b. Expenditures for exploratory drilling. c. Expenditures related to the development of mineral resources. d. Expenditures for activities in relation to evaluating the technical feasibility and commercial viability of extracting a mineral resource. 97. The accounting for a fair value hedge records the derivative at a. Amortized cost c. Fair value b. Carrying amount d. Historical cost 98. Which of the following represents the total number of shares that a corporation may issue under the terms of its charter? a. Authorized shares c. Unissued shares b. Issued shares d. Outstanding shares 99. Shares that have a fixed per-share amount printed on each share certified are called a. Stated value shares c. Uniform value shares b. Fixed value shares d. Par value shares 100.Cash dividends are paid on the basis of the number of shares a. Authorized b. Issued c. Outstanding d. Outstanding less the number of treasury shares A1 PASSERS TRAINING, RESEARCH, REVIEW AND DEVELOPMENT COMPANY// 8



A1 PASSERS TRAINING, RESEARCH, REVIEW & DEVELOPMENT COMPANY 2nd Floor Sommerset Bldg., Lopez Jaena St. Jaro, Iloilo City Tel. No.: (033) 320-2728; 09106547262 Email Address: [email protected] BOARD OF CERTIFIED PUBLIC ACCOUNTANT CERTIFIED PUBLIC ACCOUNTANT Licensure Examination SET B

AUDITING THEORY GENERAL INSTRUCTIONS: 1. This test booklet contains 80 test questions. 2. Read INSTRUCTIONS TO EXAMINEES printed on your answer sheet. 3. Shade only one (1) box for each question on your answer sheets. Two or more boxes shaded will invalidate your answer. 4. AVOID ERASURES. INSTRUCTIONS: 1. Detach one (1) answer sheet from the bottom of your Examinee ID/Answer Sheet Set. 2. Write the subject title “AUDITING THEORY” on the box provided. 3. Shade Set Box “A” on your answer sheet if your test booklet is Set A; Set Box “B” if your test booklet is Set B. __________________________________________________________________________________________ 1. An Exception in a test of control provides only an indication of the likelihood of monetary misstatements in the financial statements because test of controls do not reveal whether monetary misstatements have actually occurred. An Exception in a substantive test of transactions provides an indication of the likelihood of monetary misstatements in the financial statements because substantive tests of transaction do reveal whether monetary misstatements have actually occurred.

2.

3.

4.

5.

6.

A. True; False C. False; True B. false; False D. True; True Which of the following services provides the lowest level of assurance on a financial statement? A. An audit B. A review C. Neither service provides assurance on financial statements. D. Each service provides the same level of assurance on financial statements. Which one of the following is more difficult to evaluate objectively? A. Presentation of financial statements in accordance with PFRS. B. Compliance with government regulation. C. Efficiency and effectiveness of operations. D. All three of the above are equally difficult. If the auditor believes that the financial statements are not fairy state or is unable to reach a conclusion because of insufficient evidence, the auditor A. Should withdraw from the engagement. B. Should request an increase in audit fees so that more resources can be used to conduct the audit. C. Has the responsibility of notifying financial statement users through the auditor’s report. D. Should notify regulators of the circumstances. In comparing management fraud with employee fraud, the auditor’s risk of falling to discover the fraud is A. Greater for management fraud because mangers are inherently more deceptive than employees. B. Greater for management fraud because of management’s ability to override exiting internal controls. C. Greater for employee fraud because of the higher crime rate among blue collar workers. D. Greater for employee fraud because of the larger number of employees in the organization. Which of the following statements best describe the auditor’s responsibility regarding the detection of fraud? A. The auditor is responsible for the failure to detect fraud only when such failure clearly results from nonperformance of audit procedures specifically described in the engagement letter. B. The auditor is required to provide reasonable assurance that the financial statements are free of the both material errors and fraud. C. The auditor may extend auditing procedures to actively search for evidence of fraud where examination indicates that fraud may exits. D. The auditor is responsible for the failure to detect fraud only when an unmodified opinion is issued.

A1 PASSERS TRAINING, RESEARCH, REVIEW AND DEVELOPMENT COMPANY// 9

7. Which of the following statements is not correct? A. It is possible to vary the sample size from one unit to 100% of the items in the population. B. The decision of how many items to test should not be influenced by the increase costs of performing the additional tests. C. The decision of how many items to test must be made by the auditor for each audit procedure. D. The sample size for any given procedure is likely to vary from audit to audit. 8. For audit evidence to be compelling to the auditor it must be sufficient and appropriate. Which statement below is not correct regarding the appropriateness of audit evidence? A. The more effective the internal control system, the more assurance it provides the auditor about the reliability of financial reporting by the client. B. An auditor’s opinion to be economically useful and profitable to the auditing firm needs to be formed within a reasonable time and based on evidence obtain that assures profits for the auditing firm. C. Evidence obtained from independent sources outside the entity is generally more reliable than evidence secured solely within the entity. D. The independent auditor’s direct personal knowledge, obtained through inquiry, observation and inspection, is generally more persuasive than information obtained indirectly. 9. Which of the following is not a correct use of the terminology in relation to audit evidence? A. Evidence obtained from an independent source outside the client organization is more reliable than obtained from within. B. Documentary evidence is more reliable when it is received by the auditor indirectly rather than directly. C. Documents that originate outside the company are considered more reliable than those that originate within the client’s organization. D. External evidence, such as communications form banks is generally regarded as more reliable than answers obtained from inquires of the client. 10. When making decisions about evidence for a given audit the auditor’s goal is to obtain a sufficient amount of timely reliable evidence that is relevant to the information being verified. In addition the goal of audit efficiency is to gather and evaluate the information A. No matter the cost involved in obtaining such evidence. B. Even if cost is irrelevant to the auditor because they bill the client for costs incurred. C. At the lowest possible total cost. D. At the cost suggested in the engagement letter. 11. The auditor is concerned that a client is falling to bill customers for shipments. An audit procedure that would gather relevant evidence would be to A. Select a sample of duplicate sales invoices and trace each to related shipping documents. B. Trace a sample of shipping documents to related duplicate sales invoices. C. Trace a sample of sales Journal entries to the Accounts Receivable subsidiary ledger. D. Compare the total of the Schedule of Accounts Receivable with the balance of the Accounts Receivable account in the general ledger. 12. The purpose of an engagement letter is to A. Document the CPA firm’s responsibility to external users of the audited financial statements. B. Document the term of the engagement. C. Notify the audit staff of an upcoming engagement so that personnel scheduling can be facilitated. D. Emphasize management’s responsibility for approving the audit program. 13. The predecessor auditor is required to respond to the request of the successor auditor for information, but the response can be limited to stating that no information will be provided when A. The predecessor auditor has poor relation with the successor auditor. B. The client is dissatisfied with the predecessor’s work. C. There are actual or potential legal problems between the client and the predecessor. D. The predecessor believes that the client lacks integrity. 14. The audit team gather information about a new client’s business and industry in order to obtain A. An understanding often client’s internal control system for financial reporting. B. An understanding of how economic events and transactions have an effect on the company’s financial statements. C. Information about engagement risk. D. Information regarding whether the company is engaging in financial statement fraud. 15. During audit planning, the auditor uses analytical procedures primarily to A. Identify weakness in internal control. B. Determine if the company’s financial statements appears reasonable and are free of material misstatement. C. Determine the correspondence of the company’s financial statements to the valuation and accuracy audit objectives. D. Understand the client’s business and industry and to indicate possible misstatements. A1 PASSERS TRAINING, RESEARCH, REVIEW AND DEVELOPMENT COMPANY// 10

16. When performing planning analytical procedures for a client, the auditor detected that the gross profit percentage had declined by 50% from the previous year to the year currently under audit. The auditor should A. Investigate the possibility the client may have made an error in their cost of goods sold computation. B. Asset management in developing greater cost efficiencies in their product line. C. Prepare a going concern opinion for the client. D. Advise the client to have extensive disclosure to alleviate investor concern. 17. The auditor’s primary purpose in auditing the client’s system of internal control over financial reporting is A. To prevent fraudulent financial statements from being issued to the public. B. To evaluate the effectiveness of the company’s internal control over all relevant assertions in the financial statements. C. To report to management that the internal controls are effective in preventing misstatements from appearing on the financial statements. D. To efficiency conduct the audit of financial statements. 18. Which of the following factors may increase risks to an organization?  

Geographic dispersion of company operation Presence of new information technologies

A Yes Yes

B No No

C Yes Yes

D No No

19. Which of the following hest describes the purpose of control activities? A. The actions, policies and procedures that reflect the overall attitude management. B. The identification and analysis of risks relevant to the preparation of financial statements. C. The policies and procedures that help ensure that necessary actions are taken to address risk to the achievement of the entity’s objectives. D. Activities that deal with ongoing assessment of the quality of internal control by management. 20. A company is concerned with the theft of cash after the sale has been recorded. One way in which fraudsters conceal the theft is by a process called “lapping”. Which of the following best describes lapping? A. Reduce the customer’s account by recording a sales return. B. Write off the customer’s account C. Reduce the customer’s account by recording a sales allowance. D. Apply the payment from another customer to the customer’s account. 21. An important characteristic of IT is uniformity of processing. Therefore, a risk exits that A. Auditors will not be able to access data quickly. B. Auditors will not be able to determine if data is processed consistently. C. Erroneous processing can result in the accumulation of a great number of misstatements in a short period of time. D. All of the above. 22. Which of the following statements is not true? A. Analytical procedures emphasize the overall reasonableness of transactions and balances. B. Test of controls are concerned with evaluating whether controls are sufficiently effective to justify control risk and thereby reducing analytical procedures. C. Substantive tests of transactions emphasize the verification of transactions recorded in the journals and then posted in the general ledger. D. Tests of details of balances emphasize the ending balances in the general ledger. 23. An examination of part of an organization’s procedures and methods for the purpose of evaluating efficiency and effectiveness is what type of audit? A. Production audit. C. Compliance audit. B. Financial statement audit. D. Operation audit. 24. In many audits of the sales transactions, subtractive tests of transactions can be reduced in determining the completeness objective because A. Understatements of assets and income are a greater concern than overstatements. B. Overstatements of assets and income are a greater concern than understatements. C. It doesn’t matter if income is understated because the savings on income tax offsets the reduced revenue and net income is correct. D. The unrecorded sales cause a reduction of accounts receivable; therefore, the ratio of the two financial statements will not be misleading. 25. To determine if a sample is truly representative of the population, an auditor would be required to A. Conduct multiple samples of the same population. B Never use sampling because of the expense involved. C. Audit the entire population. D. Use systemic sample selection. A1 PASSERS TRAINING, RESEARCH, REVIEW AND DEVELOPMENT COMPANY// 11

26. The most important test of details of balances to determine the existence of recorded accounts receivable is A. Tracing detail of sales invoices to shipping documents. B. Tracing the credits in accounts receivable to bank deposit. C. Tracing sales returns entries to credit memos issued and receiving room reports. D. The confirmation of customers balance. 27. It is common to use combination of positive and negative confirmations by sending the latter to accounts with larger balances and the former to those with small balances.

28.

29.

30.

31.

32.

When sending confirmations during most audits of accounts receivable, the emphasis is often on confirming larger and older balances. A. True; True C. True; False B. false; false D. False; True Which of the following is the most effective control procedure to detect vouchers that were prepared for the payment of good that were not received? A. Count goods upon receipt in storeroom. B. Match purchase order, receiving report, and vendor’s invoice for each voucher in accounts payable department. C. Compare goods received with goods requisitioned in receiving department. D. Verify vouchers for accuracy and approval in internal audit department. Internal controls that are likely to prevent the client from including as a business expense those transactions that primarily benefit management or other employees rather than the entity being audited satisfy the control objective that A. Acquisitions are correctly valued. D. recorded acquisition are for goods or B. Existing acquisitions are recorded. services received. C. Acquisitions are correctly classified. A CPA learns that his client has paid a vendor twice for the same shipment, once based upon the original invoice and once based upon the monthly statement. A control procedure that should have prevented this duplicate payment is A. Attachment of the receiving report to the disbursement report. B. Prenumbering of disbursement vouchers. C. Use of a limit or reasonableness test. D. Prenumbering of receiving reports. If the client fails to record disposals of property, plant, and equipment, both the original cost of the asset account and the net book value will be incorrect. What will the effect be of this misstatement on the original cost and the book value? A. both will be overstated indefinitely. B. The original cost will overstated indefinitely, and the net book value will be overstated until the asset is fully depreciated. C. The original cost will be overstated indefinitely and the net book value will be understated indefinitely. D. The original cost will be overstated indefinitely and the net book value will be understated until the asset is fully depreciated. The nature, extent, and timing of substantive tests of payroll transactions vary depending, In part, on assessed control risk.

“Physical control over assets” is not a type of control that is applicable to the payroll cycle. A. True; False C. True; True B. False; True D. False; False 33. Which of the following controls would be appropriate regarding the release of materials from a stockroom? A. Production employees request materials be delivered to their work areas as they need them. B. Stockroom employees deliver materials to work areas throughout the day to maintain acceptable levels of safety stock-no written records are maintained. C. Production employees submit approved requisition forms to the stockroom for material needed. D. Production employees in needed of material should personally pick up needed materials from the Stockroom. 34. If the auditor concludes that physical controls over inventory are so inadequate that the inventory will be difficult to count the auditor should ordinary A. Withdraw from the engagement. B. Issue a qualified audit report. C. Conduct expanded observation tests of physical inventory. D. Hire an expert to assist the auditor. A1 PASSERS TRAINING, RESEARCH, REVIEW AND DEVELOPMENT COMPANY// 12

35. You are gathering evidence for the audit objective that existing inventory items are included in the inventory listing schedule. The audit procedure that would provide you with the best evidence to confirm this objective is A. trace from inventory tags to the inventory listing schedule and make sure the inventory tag is included. B. Trace the inventory totals to the general ledgers. C. Perform tests of lower-cost-or-net realizable value. D. Account for unused tags shown in the auditor’s documentation to make sure no tags have been added. 36. Which of the following is not an objective of the auditor’s examination of notes payable? A. To determine whether internal controls adequate. B. To determine whether client’s financing arrangements are effective and efficient. C. To determine whether transactions regarding the principal and interest of notes are properly authorized. D. To determine whether the liability for notes and related interest expense and accrued liabilities are properly stated. 37. Which of the following errors would be least likely to be discovered during the tests of the bank reconciliation? A. Payment was made to an employee for more hours than he worked. B. Cash received by the client subsequent to the balance sheet date was recorded as cash receipts in the current year. C. Payments on notes payable debited directly to the bank balance by the bank were not entered in the client’s records. D. Deposits were recorded in the cash receipts records near the end of the year, deposited in the bank, and were included in the bank reconciliation as a deposit in transit. 38. A major consideration in the audit of the general cash balance is the possibility of fraud. The auditor must extend his or her procedures in the audit year-end cash to determine the possibility of a material fraud when there are A. Large cash balances at the end of the year. B. Large cash receipts and disbursements during the year. C. No imprest accounts used for payroll. D. Inadequate internal controls. 39. Refusal by a client to prepare and sign the representation letter would require the auditor to issue A. Qualified opinion or a disclaimer. D. Unmodified opinion with an emphasis of B. Adverse opinion or a disclaimer. matter paragraph. C. Qualified or an adverse opinion. 40. Assurance provided by a review is substantially less than an audit. Which of the following statements is true regarding these services? A. A review requires more substantive evidence than an audit. B. An audit requires less evidence related to internal control than review. C. A review requires less evidence than an audit. D. None of the above statements is true. 41. Which of the following groups could not be involved in an operational audit? A. CPA firms. D. None is correct; that is, all of the above B. Government auditors. could be involved. C. Internal auditors. 42. Which of the following conditions most likely would pose the greatest risk in accepting a new audit engagement? A. Staff will need to be rescheduled to cover this new client. B. There will be a client-imposed scope limitation. C. The firm will have to hire an expert in one audit area. D. the client’s financial reporting system has been in place for 10 years. 43. Audit plans should be designed so that A. Most of the required procedures can be performed as interim work. B. Inherent risk is assessed at a sufficiently low level. C. The auditor can make constructive suggestion to management. D. The audit evidence gathered supports the auditor’s conclusion. 44. When expressing an unmodified opinion, the auditor who evaluates the audit findings should be satisfied that the A. Amount of known misstatement is documented in the management representation letter. B. Estimate of the total likely misstatement is less than a material amount. C. Amount of known misstatement is acknowledgement and recorded by the client. D. estimate of the total likely misstatement includes the adjusting entries already recorded by the client. A1 PASSERS TRAINING, RESEARCH, REVIEW AND DEVELOPMENT COMPANY// 13

45. In assessing the objectivity of internal auditors, an independent auditor should A. Evaluate the quality control program in effect for the internal auditors. B. Examine documentary evidence of the work performed by the internal auditors. C. Test a sample of the transactions and balances that the internal auditors examined. D. Determine the organization level to which the internal auditor report. 46. Reyes Merchandising Co. Maintains a staff of three full-time internal auditors. If the work of the internal auditors is relevant to the audit, it is efficient to consider how that work may affect the audit, and the internal auditors are found to be competent and objective, the independent auditor most likely will A. Nevertheless need to make direct tests of assertion about material financial statement amounts for which the risk of material misstatement is high. B. Decrease the extent of the tests of control needed to restrict detection risk to the acceptable level. C. Increase the extent of the procedures needed to reduce control risk to an acceptable level. D. Not evaluate and test the work performed by the internal auditors. 47. The auditor’s evaluation of the reasonableness of accounting estimates A. Should be in the context of individual transactions. B. Consider that management bases its judgement on both subjective and objective factors, C. Will be unfavourable if the estimate financial statements are based on assumption about future events and transactions. D. should be based on a attitude of conservatism. 48. A client who recently installed a new accounts payable system employees a user identification code (UIC) and a separate password. Each UIC is a person’s name, and the individuals passwords at initial log-in nor do passwords ever expire. Which of the following statements does not reflect a limitation of the client’s computer-access control? A. Employees can easily guess fellow employees passwords. B. Employees are not required to change passwords. C. Employees can circumvent procedures to separate duties. D. Employees are not required to take regular vacations. 49. A CPA is gaining an understanding of the internal controls for a client that sells garden products using an internet site. Which of the following is not likely to be found on the client’s organizational chart? A. The sales order department. C. The warehouse. B. The shipping department. D. Computer processing. 50. Which of the following describes a weakness in accounts payable procedures? A. The accounts payable clerk files invoices and supporting documentation after payment. B. The accounts payable clerk manually verifies arithmetic on the vendor invoice. C. The accounts payable system compares the receiving report to the vendor invoice. D. The accounts payable manager issues purchase orders. 51. The auditor should perform test of controls when A. Substantive procedures alone do not provide sufficient appropriate audit evidence at the relevant assertion level. B. Tests of details and substantive analytical procedures provide sufficient appropriate audit evidence to support the assertion being evaluated. C. The auditor is not able to obtain an understanding of internal controls. D. The owner-manager performs virtually all the functions of internal control. 52. An auditor who has confirmed accounts receivable may discover that the sales journal was held open past year end if A. Positive confirmations sent o debtors are not returned. B. Negative confirmations sent to debtors are not returned. C. Most of the returned negative confirmations indicate that the debtor owes a larger balance than the amount being confirmed. D. most of the returned positive confirmations indicate that the debtor owes a smaller balance than the amount being confirmed. 53. Which of the following procedures would best detect a liability omission by management? A. Inquiry of senior support staff and recently departed employees. B. Review and check mathematical accuracy of financial statements. C. Review articles of incorporation and corporate bylaws. D. Review purchase contracts and other legal documents. 54. The risk of material misstatement refers to A. Control risk and acceptable audit risk. B. Inherent risk. C. The combination of inherent risk and control risk. D. Inherent risk and audit risk.

A1 PASSERS TRAINING, RESEARCH, REVIEW AND DEVELOPMENT COMPANY// 14

55. In a financial statement audit, inherent risk is evaluated to help an auditor assess which of the following? A. the internal audit department’s objectivity in reporting a material misstatement of a financial statement assertion it detects to the audit committee. B. The risk the internal control system will not detect a material misstatement of a financial statement of a financial statement assertion. C. The risk that the audit procedures implemented will not detect a material misstatement of a financial statement assertion. D. The susceptibility of a financial statement assertion to a material misstatement assuming there are no related controls. 56. Auditors begin their assessments of inherent risk during audit planning. Which of the following would not help in assessing inherent risk during the planning phase? A. Obtaining client’s agreement on the engagement letter. B. Obtaining knowledge about the client’s business and industry. C. Touring the client’s plant and offices. D. Identifying related parties. 57. As the acceptable audit risk is decreased, the likely cost of conducting an audit increases.

58.

59.

60.

61.

62.

63.

64.

Acceptable audit risk is measure of the auditor’s willingness to accept that the financial statements do not contain material misstatements after the audit is completed and a qualified audit report has been issued. A. True; True C. False; False B. True; False D. False; True Related party transactions may be indicated when another company A. Subsidizes certain operating expense of the company. B. Purchases its securities at their fair value. C. loans to company at market rates. D. Has had a distributor relationship with the company for 10 years. An auditor should examine minutes of the board of directors’ meetings A. Through the date of the financial C. Only at the beginning of the audit. statements. D. On a test basis. B. Through the date of the audit report. If the auditor were responsible for making certain that all of management’s assertions is the financial statements were absolutely correct A. Bankrupts could no longer occur. B. Bankrupts would be reduced to a very small number. C. Audits would not be much easier to complete. D. Audits would not be economically practical. Which of the following statements is true? A. Auditors have generally found that the most effective and efficiently way to conduct an audit is to obtain some assurance for each class of transactions and for the ending balance of the related account. B. Management’s assertion follow and are closely related to the audit objectives. C. The auditor’s primary responsibility is to find and disclose fraudulent management assertions. D. Assertions about presentation and disclosure deal with whether the accounts have been included in the financial statements at appropriate amounts. Which of the following statements about the existence and completeness assertions is not true? A. The existence and completeness assertions emphasize different concerns. B. Existence deals with overstatements and completeness deals with understatements. C. Existence deals with understatements and completeness deals with overstatements. D. The completeness assertion deals with unrecorded transactions. In testing for cutoff, the objective is to determine A. Whether all of the current period’s transactions are recorded. B. Whether transactions are recorded in the correct accounting period. C. The proper cutoff between capitalizing and expensing expenditures. D. The proper cutoff between disclosing items in notes to the financial statements or in accounts balances. Determine which of the following is most correct regarding the reliability of audit evidence? A. Information that is indirectly obtained from external sources is the most reliable audit evidence. B. Reliability of audit evidence is dependent upon the evidence being convincing. C. Reliability of evidence refers to the amount of evidence obtained. D. An effective control system provides more reliable audit evidence.

A1 PASSERS TRAINING, RESEARCH, REVIEW AND DEVELOPMENT COMPANY// 15

65. When the auditor develops supporting evidence for amounts posted to account balances with documentary evidence, that process is called A. Inquiry. C. Vouching. B. Confirmation. D. Physical examination. 66. Narratives, flowcharts, and internal control questionnaires are three common methods of A. Documenting the auditors understanding of internal controls. B. Designing the audit manual and procedures. C. Documenting the auditor’s understanding of a client’s organization structure. D. Testing the internal controls. 67. Who is most likely to perpetrate fraudulent financial reporting? A. Members of the board of directors. C. Management of the company. B. Production employees. D. The internal auditors. 68. Which of the following is not a factor that relates to opportunities to misappropriate assets? A. Inadequate internal controls over assets. B. Presence of large amounts of cash on hand. C. Inappropriate segregation of the duties or independent checks on performance. D. Adverse relationships between management and employees. 69. A control that relates to all parts of the IT system is called a/an A. General controls. C. Universal control. B. Systems control. D. Applications control. 70. The auditor would design which of the following audits test to detect possible monetary errors in the financial statement? A. Controls tests. D. Test of operating effectiveness of B. Analytical procedures. controls over revenue and cash. C. Risk assessment procedures. 71. If inherent risk is increased to medium from low, test of details of balances can be reduced.

72. 73.

74.

75.

76.

The extent of test of details of balances cannot be reduced when transaction-related audit objectives have been satisfied by tests of controls or substantive tests of transactions. A. True; False C. True; True B. False; True D. False; False A document that details what the auditor will do to gather sufficient, appropriate evidence is the A. Audit strategy. C. Audit procedure B. Audit program D. Audit risk model Analytical procedures must be performed in A. The planning and test of control stages. B. Conjunction with tests of transactions and tests of details of balance. C. The planning and completion stages. D. The planning, test of control, and completion stages. An auditor noted that the accounts receivable department is separate from other accounting activities. Credit is approved by a separate credit department. Control accounts and subsidiary ledgers are balanced monthly. Similarly, accounts are aged monthly. The accounts receivable manager writes off delinquent accounts after 1 year, or sooner if a bankruptcy or other or other unusual circumstances are involved. Credit memoranda are prenumbered and must correlate with receiving reports. Which of the following areas could be viewed as an internal control weakness of the above organization? A. Write-offs of delinquent accounts. C. Monthly aging of receivables. B. Credit approvals. D. Handling of credit memos. A purchasing agent places an order for inventory whenever a requisition is received from the warehouse. The warehouse clerk issues requisitions based on predict physical counts because no perpetual records are maintained. Numerous duplicate orders have been placed for goods previously ordered but not received. To prevent this excess ordering, the firm should A. Keep an adequate record of open purchase orders and review it before ordering. B. Count goods in the warehouse less often. C. Use prenumbered purchase orders. D. Not use purchase requisition. Negative confirmation of accounts receivable is less effective than positive confirmation of accounts receivable because A. A majority of recipients usually lack the willingness to respond objectively. B. some recipients may report incorrect balances that require extensive follow-up. C. The auditor cannot infer that all nonrespondents have verified their account information. D. Negative confirmations do not produce evidence that is statistically quantifiable.

A1 PASSERS TRAINING, RESEARCH, REVIEW AND DEVELOPMENT COMPANY// 16

77. The refusal of a client’s lawyer to provide a representation on the legality of a particular act committed by the client is ordinarily A. Sufficient reason to issue a “Subject to” opinion. B. Considered to be a scope limitation. C. Insufficient reason to modify the auditor’s report because of the lawyer’s obligation of confidentiality. D. Proper grounds to withdraw from the engagement. 78. Which of the following steps should an auditor perform first to determine the existence of related parties? A. Examine invoices, contracts, and purchase orders. B. Request a list of related parties from management. C. Review the company’s business structure. d. Review proxy and other materials filed with the SEC. 79. In planning the audit engagement, the auditor should consider each of the following except A. The auditor’s independence. B. Risk of material misstatement due to fraud. C. Anticipated levels of audit risk and materiality. D. The kind of opinion (unmodified, qualified, or adverse) that is likely to be expressed. 80. After obtaining a sufficient understanding of the entity, its environment, and internal control in an audit of the financial statements, the auditor assesses. A. The need to apply auditing standards. B. Detection risk determining the acceptable level of inherent risk. C. Detection risk and inherent risk to determine the acceptable level of control risk. D. Control risk and inherent risk to determine the acceptable level of detection risk.

A1 PASSERS TRAINING, RESEARCH, REVIEW AND DEVELOPMENT COMPANY// 17



A1 PASSERS TRAINING, RESEARCH, REVIEW & DEVELOPMENT COMPANY 2nd Floor Sommerset Bldg., Lopez Jaena St. Jaro, Iloilo City Tel. No.: (033) 320-2728; 09106547262 Email Address: [email protected] BOARD OF CERTIFIED PUBLIC ACCOUNTANT CERTIFIED PUBLIC ACCOUNTANT Licensure Examination SET A

MANAGEMENT ADVISORY SERVICES GENERAL INSTRUCTIONS: 1. This test booklet contains 50 test questions. 2. Read INSTRUCTIONS TO EXAMINEES printed on your answer sheet. 3. Shade only one (1) box for each question on your answer sheets. Two or more boxes shaded will invalidate your answer. 4. AVOID ERASURES. INSTRUCTIONS: 1. Detach one (1) answer sheet from the bottom of your Examinee ID/Answer Sheet Set. 2. Write the subject title “MANAGEMENT ADVISORY SERVICES” on the box provided. 3. Shade Set Box “A” on your answer sheet if your test booklet is Set A; Set Box “B” if your test booklet is Set B. __________________________________________________________________________________________

USE THE FOLLOWING TO ANSWER QUESTION 1-4: Little Star Products Company has a maximum productive capacity of 100,000 units per year. Normal capacity is 90,000 units per year. Standard variable manufacturing cost are P20 per unit. Fixed factory overhead is p450,000 per year. Variable selling expense is P10 per unit, and fixed selling expense is P300,000 per year. The unit sales price is P50. The operating results for the year are as follows: sales, 80,000 units; and beginning inventory, 5,000 units. All variances are written off as additions (or deductions from) the standard cost of goods sold. 1. What is the break-even point expressed in peso sales? a. P1,125,000 c. P1,250,000 b. P1,875,000 d. P 37,500 2. How many units must be sold to earn a net income of P50,000 per year? a. 2 million c. 40,000 b. 37,500 d. 25,000 3. Income under absorption costing is: a. P900,000 c. P 925,000 b. P975,000 d. P1,975,000 4. Income uncer variable costing is: a. P 25,000 c. P825,000 b. P875,000 d. P850,000 5. Product costs appear on the balance sheet: a. only if goods are partially completed at the end of the period. b. only if goods are unsold at the end of a period. c. only if partially completed or are end of a period. d. only in merchandising firms. 6. Data for A and Cost B are as follows: Number of Units Produced Unit Cost Total Cost COST A 1 ? P10 10 ? P100 100 ? P1,000 1,000 ? P10,000 COST B 1 P5,000 ? 10 P500 ? 100 P50 ? 1,000 P5 ?

A1 PASSERS TRAINING, RESEARCH, REVIEW AND DEVELOPMENT COMPANY// 18

Which of the above best describes the behaviour of Cost A and B? a. Cost A is fixed, Cost B is variable. c. Both Cost A and Cost B are variable. b. Cost A is variable, Cost B is fixed. d. Both Cost A and Cost B are fixed. 7. Differential cost can: a. Only be fixed costs. c. Be either fixed or variable. b. Only be variable costs. d. Be incremental but not decremental. 8. If Carne Company were to sell 32,000 units, the total expected cost per units would be: a. P75,000 c. P80,000 b. P78,000 d. P77,000 9. If Carne Company were to sell 40,000 units, the total expected cost per unit would be: a. P2.50 c. P2.13 b. P2.25 d. P1.88 10. John Johnson decided to leave his former job where he earned P12 per hour to go to a new job where he will earn P13 per hour. In the decision process, the former wage of P12 per hour would be classified as a(n): a. Sunk cost. c. Fixed cost. b. Direct cost. d. Opportunity cost. 11. The Standards of Ethical Conduct for Management Accountants developed by the Institute of Management Accountants contain a policy regarding confidentially that requires management accountants to refrain from disclosing confidential information acquired in the course of their work: a. Except when authorized by management. b. In all situations. c. Except when authorized by management, unless legally obligation to do so. d. In all cases not prohibited by law. 12. During the month of January, direct labor cost totalled P17,000 and direct labor cost was 60% of prime cost. If total manufacturing costs during January were P82,000 the manufacturing overhead was: a. P11,333 c. P28,333 b. P53,667 d. P65,000 13. Samantha Galloway is a managerial accountant in the accounting department of Mustang Industries, Inc. Samantha has just discovered evidence that some of the corporation’s marking mangers have been wrongfully inflating their expense reports in order to obtain higher reimbursements from the firm . According to the Institute of Management accountants’ Standards of Ethical Conduct, what should Samantha do upon discovering this evidence? a. notify the controller. b. notify the marking managers involved. c. notify the president of the corporation d. Ignore the evidence because she is not part of the Marking Department. 14. Which of the following statements is true when referring to fixed cost? a. Committed fixed costs arise from the annual decisions by management. b. As volume increases, unit fixed cost and total fixed cost will change. c. Fixed costs increase in total throughout the relevant range d. Discretionary fixed costs can often be reduced to zero for short periods of the without seriously impairing the long-run goals of the company 15. A P2.00 increase in a product’s variable expense per unit accompanied by a P2.00 Increase in its selling price per unit will: a. decrease the degree of operating leverage. b. decrease the contribution margin. c. have no effect on the break-even volume (In units). d. have no effect on the contribution margin ratio. 16. The following information relates to the break-even point at Pezzo Corporation: Sales pesos P120,000 Total fixed expenses P30,000 If Pezzo wants to generate net operating income of p12, 000, what will its sales pesos have to be? a. P132,000 c. P168,000 b. P136,000 d. P176,000 17. The contribution approach income statement: a. Organizes costs on a function basis. b. Provides owners with more cash flows. c. Is particularly helpful to the manager in planning and decision making. d. Provides a gross margin figure from which selling and administrative expenses are deducted.

A1 PASSERS TRAINING, RESEARCH, REVIEW AND DEVELOPMENT COMPANY// 19

18. Minist company sells a single product at a selling price of p15.00 per unit. Last year, the company’s sales revenue was P225, 000 and its net operating income was P18,000.

19.

20.

21. 22.

23.

If fixed expenses totalled P72,000 for the year, the break-even point in unit sales was a. 15,000 c. 14,100 b. 9,000 d. 12,000 A single- product company prepares income statements using both absorption and variable costing methods. Manufacturing overhead cost applied per unit produced under absorption costing in year 2 was the same as in year 1. The year 2 variable costing statement reported a profit whereas the year 2 absorption costing statement reported a loss. The difference in reported income could be explained by units produced in year 2 being: a. Less than units sold in year 2. b. Less than the activity level used for allocating overhead to the product. c. In excess of the activity level used for allocating overhead to the product. d. In excess of units sold in year 2 Which of the following costs should not be included in product costs for internal management reports that are used for decision-making? a. Cost of unit-level activities. d. Costs of organization-sustaining b. Costs of batch-level activities. activities c. Costs of product-level activities. If all sub-variances are calculated for labor, which of the following cannot be determined? a. Labor rate variance. c. Reason for the labor variances. b. Actual hours of labor used. d. Efficiency of the labor force. Under variable costing: a. Net operating income will tent to move up and down in response to changes in levels of production. b. Inventory costs will be lower than under absorption costing. c. Net operating cost income will tend to vary inversely with production changes. d. Net operating income will always be higher than under absorption costing. Hettich Corporation uses an activity-based costing system with the following three activity cost pools:

Activity Cost Pool Total Activity Fabrication 20,000 machine-hours Order processing 200 orders Other Not applicable The other activity cost pool is used to accumulate costs of Idle capacity and organization- sustaining costs. The company has provided the following data concerning its costs: Wages and salaries P480,000 Depreciation 120,000 Occupancy 200,000 Total P800,000 The distribution of resource consumption across activity cost pools is given below:

Wages and salaries Depreciation Occupancy

Fabrication 55% 10% 25%

Order processing 20% 45% 40%

Other 25% 45% 35%

Total 100% 100% 100%

The activity rate for the Order processing activity cost pool is closest to: a. P1,400 per order c. P1,150 per order b. P1,600 per order d. P800 per order 24. The following information relates to Snowbird Corporation: Sales at the break-even point Total fixed expenses Net operating income

P312,500 P250,000 P150,000

What is snowbird’s margin of safety? a. P62,500 c. P100,000 b. P187,500 d. P212,500 A1 PASSERS TRAINING, RESEARCH, REVIEW AND DEVELOPMENT COMPANY// 20

25. Which of the following is true of a company that uses absorption costing? a. Net operating income fluctuates directly with changes in sales volume. b. Fixed production and fixed selling costs are considered to be product costs. c. Unit product costs can change as a result of changes in the number of units manufactured. d. Variable selling expenses are included in product costs. USE THE FOLLOWING TO ANSWER QUESTIONS 26-30: The Dresden Company uses standard costing for the single product the company makes and sells. The following data are for the month of April:  Actual cost of direct material purchased and used: P62,400  Material price variance: P4,800 unfavourable  Total materials variance: P14,400 unfavourable  Standard cost per pound of material: P6  Standard cost per direct labor hour: P8  Actual direct labor hours: 3,800 hours  Labor efficiency variance: P1,600 favourable  Standard number of direct labor hour per unit of product: 2  Total labor variance: P680 unfavourable 26. The total number of units produced during April was: a. 8,000 c. 2,000 b. 12,000 d. 3,800 27. The standard quality of material allowed to produce one unit of product was: a. 1 pound c. 6 pounds b. 4 pounds d. 2 pounds 28. The actual material cost per pound was: a. P6.50 c. P5.00 b. P6.00 d. P7.20 29. The actual direct labor rate per hour was: a. P16.00 c. P8.00 b. 6.50 d. 8.60 30. The labor rate variance was: a. P2,280 favourable c. P920 favourable b. P2,280 unfavourable d. P920 unfavourable 31. The nut house, Inc, sells three types of nuts: almonds, cashews, and walnuts. Ten thousand cans of nuts were sold in 2011, and the amount of walnuts sold were twice as much as the number of cans of cashews, whereas almond sakes were one-half the amount of cashew sales. Fixed costs were P37,680, and the unit variable costs were as follow: Product Almond Cashews Walnuts

Unit Sales Price P8 10 6

Unit Variable Cost P4 5 4

The company plans to earn profit of P6,280. The overall break-even unit sales is: a. 10,000 c. 12,000 b. 14,000 d. 6,857 32. Qualitative factors: a. Generally are easily measured quantitative terms. b. Are generally irrelevant for decision making. c. May include either financial or nonfinancial information d. Include customer satisfaction. 33. Lacob Corporation is a wholesaler that sells a single product. Management has provided the following cost data for two levels of monthly sales volume. The company sells the product for P103.40 per unit. Sales volume (units) 5,000 6,000 Cost of sales P315,500 P378,000 Selling, general, and administrative cost P162,500 P177,600 The best estimate of the total contribution margin when 5,300 units are sold is: a. P 56,710 c. P 41,340 b. P133,560 d. P213,590 A1 PASSERS TRAINING, RESEARCH, REVIEW AND DEVELOPMENT COMPANY// 21

34. Of the following which is the best reason for using activity-based costing? a. To keep better track of overhead costs b. To more accurately assign overhead costs to cost pools so these cost are better controlled. c. To better assign overhead costs to products. d. To assign indirect service overhead costs to direct overhead cost pools. 35. Stead Company produces a single product. Last year, the company’s net operating income computed by the absorption costing method was P6,400, and its net operating income computed by the variable costing and method was P9,100. The company’s unit product cost was P17 under variable costing and P20 under absorption costing. If the ending inventory consisted of 2,100 units the beginning inventory in units must have been: a. 1,200 c. 3,000 b. 2,100 d. 4,800 36. Black Tool Company has a production capacity of 1,500 units per month, but current production is only 1,250 units. The manufacturing costs are P60 per unit and marking costs are P16 per unit. Doug Hall offers to purchase 250 units at P76 each for the next five months. Should Black accept the one-timeonly special order if only absorption-costing data are available? a. Yes, good customer relations are essential. b. No, the company will only break even. c. No, since only the employees will benefit. d. Yes, since operating profits will most likely increase. 37. If Konrade’s engine Company purchases 1,000 TE456 parts from the outside supplier per month, then its monthly avoidable costs (costs that will no longer be incurred) total: a. P 82,000 c. P 50,000 b. P 98,000 d. P100,000 38. If Konrade’s Engine Company purchases 1,000 Te456 parts from the outside supplier per month, then its monthly operating income will: a. Increase by P2,000 c. Decrease by P3,000 b. Increase by P80,000 d. Decrease by P85,000 39. The maximum price that Konrade’s Engine Company should be willing to pay the outside supplier is: a. P80 per TE456 part c. P98 per TE456 part b. P82 per TE456 part d. P100 per TE456 part 40. The sum of the material price variance (calculated at point of purchase) and material quality variance equals. a. The total cost variance. c. The material mix variance. b. The material yield variance d. No meaningful number. 41. In activity-based costing, final cost allocations assign costs to a. Departments c. Products b. Processes d. Activities 42. King Company produces a single product. During March, the company had net operating income under absorption costing that was P3,500 lower than under variable costing. The company sold 7,000 units in March, and its variable costs were P7 per unit of which P3 was variable selling expense. If fixed manufacturing overhead was P2 per unit under absorption costing then how many units did the company produce during March? a. 5,250 units c. 6,500 units b. 8,750 units d. 6,125 units 43. Engagements should be adequately planned, supervised, and controlled. Controlling involves the measurement of progress in attaining the engagement plan and objectives. At significant engagement points, progress should be measured in item of a. Time schedule, accomplishments, and quality of work. b. Accomplishment, time schedule, and expenses incurred. c. Quality of work, number of reports prepared, and time schedule. d. Accomplishments, number of personnel who played a role in the engagement and attendance of the participants in the engagement. 44. A company that produces a single product had a net operating income of P85,500 using variable costing and a net operating income of P90,000 using Absorption costing. Total fixed manufacturing overhead was P150, 000 and production was 100,000 units. Between the beginning and the end of the year, the inventory level: a. Increase by 4,500 units c. Increase by 3,000 units b. Decrease by 4,500 units d. Decrease by 3,000 units 45. Which of the following is least likely to be considered an advantage of a database? a. Easy to store large qualities of information. b. Easy to retrieve information quickly. c. Easy to organized and reorganize information. d. Easy to distribute information to every possible user. A1 PASSERS TRAINING, RESEARCH, REVIEW AND DEVELOPMENT COMPANY// 22

46. Which of the following market features is likely to cause a surplus of a particular product? a. A monopoly c. A price ceiling b. A price floor d. A perfect market 47. All of the following are ways that companies in developed countries generally amy complete with companies in developing countries except a. Technology. c. Quality b. Customer service. d. Low-cost resources. 48. Laptop computers provide automation outside of the normal office location. Which of the following would provide the least security for sensitive data stored on a laptop computer? a. Encryption of data files on the laptop computer. b. Setting up a password for the screensaver program on the laptop computer. c. Using a laptop computer with a removable hard disk drive. d. Using a locking device that can secure the laptop computer to an immovable object. 49. Accounting systems are designed to attain some objectives. Which of the following cannot be considered an objective of accounting systems? a. To provide a means by which interested parties may be given information on the financial position and results of operations of a business organization. b. To protect the business owners safeguard their assets. c. To facilitate management planning control and decision-making. d. To accomplish the different routinary administrative activities of the business organization. 50. The following information relates to Zinc Corporation for last year: Sales Net operating income Degree of operating leverage

P500,000 25,000 5

Sales at Zinc are expected to be P600,000 next year. Assuming no change in cost structure, This means that net operating income for next year should be: a. P30,000 b. P45,000 c. P50,000 d. P125,000

A1 PASSERS TRAINING, RESEARCH, REVIEW AND DEVELOPMENT COMPANY// 23



A1 PASSERS TRAINING, RESEARCH, REVIEW & DEVELOPMENT COMPANY 2nd Floor Sommerset Bldg., Lopez Jaena St. Jaro, Iloilo City Tel. No.: (033) 320-2728; 09106547262 Email Address: [email protected] BOARD OF CERTIFIED PUBLIC ACCOUNTANT CERTIFIED PUBLIC ACCOUNTANT Licensure Examination SET A

AUDITING PROBLEMS GENERAL INSTRUCTIONS: 1. This test booklet contains 60 test questions. 2. Read INSTRUCTIONS TO EXAMINEES printed on your answer sheet. 3. Shade only one (1) box for each question on your answer sheets. Two or more boxes shaded will invalidate your answer. 4. AVOID ERASURES. INSTRUCTIONS: 1. Detach one (1) answer sheet from the bottom of your Examinee ID/Answer Sheet Set. 2. Write the subject title “AUDITING PROBLEMS” on the box provided. 3. Shade Set Box “A” on your answer sheet if your test booklet is Set A; Set Box “B” if your test booklet is Set B. __________________________________________________________________________________________

PROBLEM NO.1 Anyang corp. invested its excess cash in marketable equity securities during 2012. The securities do not qualify as financial asset held for trading. Anyang Corp. Has made an irrevocable election to present in other comprehensive income subsequent changes in fair value of its investment securities. As of December 31,2012 the company’s securities portfolio consisted of the following: Investee Company Kandong, inc. Egoy corp. Yoga Enterprises Totals

Share 30,000 60,000 60,000

Cost P1,350,000 4,500,000 6,480,000 P12,330,000

Fair Value P1,275,000 4,830,000 6,900,000 P13,005,000

During the year 2013, Anyang Corp. Sold 60,000 shares of Egoy Corp. For P4,800,000 and purchased 60,000 addition shares of kingdom, Inc. And 30,000 shares of Company. On December 31,2013, Anyang Corp’s portfolio of securities comprised the following: Investee Company Kandong, Inc. Kandong, Inc. Kongga Company Yoga Enterprises Totals

Shares 30,000 60,000 30,000 60,000

Cost P1,350,000 3,900,000 1,560,000 6,480,000 P13,290,000

Fair Value P1,500,000 4,350,000 1,440,000 2,100,000 P9,390,000

During the year 2014, Anyang Corp. Sold all the Kandong, Inc. Share. Also, 15,000 shares of Kongga Company were sold at a loss of P270,000. The net realized gain on sale of securities in 2014 amounted to P1,440,000. On December 31,2014, Anyang Corp’s portfolio of securities consisted of the following: Page 1 AUDITING PROBLEMS Investee Company Shares Cost Fair Value Yoga Enterprises 60,000 P6,480,000 P12,600,000 Kongga Company Totals

15,000

780,000 P7,260,000

540,000 P13,140,000

1. For the year ended December 31,2013, Anyang’s statement of comprehensive Income should report unrealized loss of A. P1,3,000,000 C. P3,915,000 B. P1,190,000 D. P4,245,000 A1 PASSERS TRAINING, RESEARCH, REVIEW AND DEVELOPMENT COMPANY// 24

2. What amount of unrealized loss should be reported in Anyang’s December 31,2013 Statement of Changes in Equity? A. P3,240,000 C. P4,245,000 B. P3,570,000 D. P3,900,000 3. How much was received by Anyang from the sale of its investment in Kandong securities In 2014? A. P6,960,000 C. P7,020,000 B. P7,560,000 D. P8,280,000 4. Anyang’s statement of Financial Position should report investment in equity securities of Dec. 31,2014 Dec. 31,2013 Dec. 31,2012 A. P12,330,000 P13,290,000 P7,260,000 B. 13,005,000 9,390,000 13,140,000 C. 12,330,000 9,390,000 7,260,000 D. 13,005,000 13,290,000 13,140,000 5. What amount should be reported as unrealized gain in Anyang’s Statement of Changes in Equity for 2014? A. P6,420,000 C. P6,960,000 B. P5,880,000 D. P14,760,000 PROBLEM NO. 2 DARWIN CORP., a manufacturer of computer part5s, has been experiencing growth in the demand for its products over the last several years. This prompted the company to obtain addition manufacturing. A real estate firm located an available factory and used machinery from production facility and Darwin agreed to purchase the factory and used machinery from Quezon Company on October 1, 2011. Renovations were necessary to convert the factory for Darwin’s manufacturing use. The terms of the agreement required Darwin to pay Quezon P4,500,000 when renovations started on January 1,2012, with the balance to be paid as renovations were complete. The overall purchase price for the factory and machinery was P36,000,000. The building renovations were contracted to Mallbornes Construction company at P9,000,000. The payments made as renovation progressed during 2012 are shown below. The factory was placed in service on January 1, 2013.

January 1 April 1 October 1 December 1

QUEZON P 4,500,00 8,100,000

MALLBORNES P 2,700,000 2,700,000

9,900,000 13,500,000 P36,000,000

3,600,000 3,600,000 P9,000,000

On January 1, 2012, Darwin obtained a 2-year, P9 million loan with 12% interest rate to finance the renovation of the acquired factory. This is Darwin’s only outstanding loan during 2012. Darwin’s policy regarding purchases of this nature is to use the appraisal value of the land for book purposes and prorate the balance of the purchase price over the remaining items. The building had originally cost Quezon P27,000,000 and had a net book value of P4,500,000 , while the machinery originally cost P11,250,000 and had a net value of P3,600,000 on the date of sale. The land was recorded on Quezon’s books at 3,600,000. The following values were determined based on appraisal conducted by independent appraisers at the time acquisition. Land P26,100,000 Building 9,450,000 Machinery 4,050,000 Gino G. Nario, Darwin’s chief engineer estimated that the renovated plant would be used for 15 years, with an estimate residual value of P27,700,000. Nario estimated that the productive machinery would have a remaining useful life of 5 years and residual value of P270,000. Darwin’s depreciation policy is to apply the 200% declining balance method for machinery and the 150% declining balance method for the plant. One-half year’s depreciation is taken in the year the plant is placed in service and one-half is allowed when the property is disposed of or retired.

A1 PASSERS TRAINING, RESEARCH, REVIEW AND DEVELOPMENT COMPANY// 25

Determined the amounts to be recorded on the books of Darwin corp. As of December 31, 2012, for each of the following properties. 6. Land A. P23,727,000 B. P27,180,000 7. Building A. P17,010,000 B. P18,670,800 8. Machinery A. P3,681,900 B. P3,294,000

C. P26,100,000 D. P32,272,000 C. P9,670,800 D. P15,930,000 C. P4,005,900 D. P2,970,000

Calculate the December 31,2014 net book value of the following: 9. Building A. P14,312,700 B. P15,351,525 10. Machinery A. P1,425,600 B. P1,069,200

C. P13,778,100 D. P14,543,550 C. P1,296,000 D. P1,188,000

PROBLEM NO.3 Presented below are three(3) Independent cases relating to the audit of shareholders’ equity. Answer the question/s at the end of each case. 1. KANDABA COMPANY began operations on January 1, 2012, by Issuing at P15 per share one-half of the 480,000 ordinary shares (P1 par value) that had been authorized for issue. In addition, Kandaba has 250,000 6% preference shares (P5 par value) authorized. During 2012, Kandaba reported net income of P735,00 and declared dividends of P112,250. During 2013, Kandaba completed the following transactions: Jan. 10 Apr. 2 July 21

Issued an additional 35,000 ordinary shares for P16.50 per share. Issued 80,000 preference shares for P7 per share. Authorized the acquisition of a custom-made machine to be delivered in January 2014. Kandaba appropriated p147,500 of retained earnings for the purchase of the machine. Oct. 25 Issued an addition 20,000 preference shares for P10 per share. Dec. 31 Reported P732,000 of net income and declared a dividend of P420,000 to shareholders of Record on January 31,2014, to be paid on February 4,2014. 11. What is the total shareholders’ equity on December 31, 2013? A. P4,222,750 C. P5,724,750 B. P5,872,250 D. P6,019,750 12. What is the unappropriated retained earnings balance on December 31, 2013? A. P4,807,250 C. P4,534,750 B. P4,222,750 D. P4,387,250 2.BULDOG CO. Is authorized to issue 300,000 of P2 par value ordinary shares. The company has the following transactions: a) Issued 60,000 shares at P32 per share; receive cash. b) Issued 1,000 shares, selling at P36 per share, to lawyers for services in connection with the organization of the corporation. The value of the legal services was P35,000. c) Issued 1,500 shares, valued objectively at P33,000, to the employees instead of paying them cash wages. d) Issued 137,500 shar5es in exchange for a building value at P885,000 and land valued at P24,000. (The building was originally acquired by the investor for P750,000 and has P300,000 of accumulated depreciation; the land WAS ORIGINALLY acquired for P90,000.) e) Received cash for 29,500 shares issued at P38 per share. 13. The statement of financial position will rep[ort share premium of A. P3,777,000 C. P3,775,000 B. P5,675,000 D. P3,776,000 A1 PASSERS TRAINING, RESEARCH, REVIEW AND DEVELOPMENT COMPANY// 26

3. TAGOGO COMPANY has been paying regular quarterly dividends of P1.50 and wants to pay the same amount in the third quarter of 2013. The following information relates to the company’s equity: Jan. 1 Feb. 15 Mar. 31 May 12

Shares outstanding, 500,000; P2 par (900,000 shares authorized). Issued 30,000 new shares at P10.50. Paid quarterly dividends of P1.50 per share. Converted P1,000,000 bonds tom ordinary shares at the rate of 50 shares per P1,000 bond. June 15 Issued a 10% share dividend. 30 Paid quarterly dividends of P2 per share. 14. What is the total amount that Tagogo will have to pay in dividends in the third quarter in order to pay P2 per share? A. P1,276,000 C. P1,260,000 B. P1,160,000 D. P1,060,000 15. What is the total amount of dividends to be distributed during the year assuming no equity transactions occur after June 30? A. P4,640,000 C. P4,240,000 B. P5,104,000 D. P4,888,000 PROBLEM NO.4 Presented below are the condensed income statements of LAROSA CORPORATION for the years Ended December 31,2013 and 2012.

Sales Cost of goods sold Gross Income Operating expenses Operating income Gain on sale of division Income tax expenses (30%) Net Income

2013 P7,500,000 5,025,000 2,475,000 1,012,500 1,462,500 300,000 1,762,500 528,750 P1,233,750

2012 P7,350,000 4,950,000 2,400,000 975,000 1,425,000 --1,425,000 427,500 P997,500

On October 10, 2013, Larosa entered into an agreement to sell the assets of one of its geographical segments. The geographical segment comprises operations and cash flows that can be clearly distinguished operationally and for financial reporting purposes, from the rest of the company. The segment was sold on December 31, 2013, for P2,625,000. The book value of the of the segment’s assets was P2,325,000. The segment’s contribution to Larosa’s operating income before tax for each for year was as follows: 2013 P170,625 loss 2012 P121,875 income 16. What amount should be reported as income from continuing operations for 2012? A. P912,187 C. P937,782 B. P997,500 D. P1,122,188 17. What amount should be reported as income from continuing operations for 2013? A. P904,312 C. P1,143,187 B. P933,188 D. P1,233,750 18. What amount should be reported as income (loss) from discontinued operations for 2013? A. P170,625 C. (P119,438) B. P210,000 D. P90,562 19. Assume that by December 31,2013, the segment had not yet been sold but was considered held for sale. The fair value of the segment’s assets on December 31 was P2,625,000. The post-tax income (loss) from discontinued operations for 2013 should be A. (P90,562) C. 90,562 B. 119,438 D. (P119,438) 20. Assume that by December 31,2013, the segment had not yet been sold but was considered held for sale. The fair value of the segment’s assets on December 31 was P1,875,000. The post-tax income (loss) from discontinued operations for 2013 should be A. P90,562 C. (P119,438) B. P1,193,062 D. (P434,438) A1 PASSERS TRAINING, RESEARCH, REVIEW AND DEVELOPMENT COMPANY// 27

PROBLEM NO.5 ACERO CORP. Was Incorporated on January 2, 2012. The corporation’s financial statements for its first year’s operations were not examined by a CPA. You have been engaged to audit the financial statements for the year ended December 31, 2013, and your audit is substantially completed. The corporation’s trial balance appears below. Acero Corp. TRIAL BALANCE December 31,2013 Debit P 450,000 2,190,000

Cash Accounts receivable Allowance for doubtful accounts Inventories Machinery and equipment Accumulated depreciation Patents Leasehold improvements Prepaid expenses Goodwill Licensing agreement No.1 Licensing agreement No.2 Accounts payable Unearned revenue Share capital Retained earnings, January 1,2013

Credit

P43,800 1,506,000 3,570,000 786,000 3,846,000 900,000 1,350,000 900,000 1,800,000 1,680,000 2,190,000 518,400 9,000,000 4,771,800

Sales Cost of goods sold Selling and administrative expenses Interest expense Loss on extinguishments of dept Totals

21,600,000 14,250,000 5,583,000 285,000 600,000 P38,910,000

P38,910,000

The following information relates to accounts that may yet require adjustment. 1. Patents for Acero’s manufacturing process were acquired January 2, 2013, at a cost of P2,805,000. An additional P1, 041,000 was spent on December 29,2013, to improve machinery covered by the patents and charged to the Patents account. Depreciation on property, plant, and equipment has been properly recorded for 2013. Acero uses the straight-line method for all depreciation and amortization and the legal life on its patents. 2. On January 3, 2012, Acero purchased Licensing Agreement No.1, which was believed to have an indefinite useful life. The balance in the Licensing Agreement No.1 account includes its purchase price of P1, 710,000 and expenses of P90,000 related to the acquisition. On January 1,2013, Acero purchases Licensing Agreement No.2, which has a life expectancy of 10 years. The balance in the Licensing Agreement No.2 account includes its P1, 620,000 purchase price and P180,000 in acquisition expenses, but it has been reduced by a credit of P120,000 for the advance collection of 2014 revenue from the agreement. In late December 2012, an explosion caused a permanent reduction in the expected revenue-producing value of Licensing Agreement No.1 and in January 2014, a flood caused additional damage that rendered the agreement worthless. The recoverable amount of Licensing Agreement No.1 was determined to be P720, 000 at December 31, 2012. 3. The balance in the Goodwill account represents amount paid on December 30, 2012, for a four-year advertising program, estimated to assist in creasing Acero’s sales.

A1 PASSERS TRAINING, RESEARCH, REVIEW AND DEVELOPMENT COMPANY// 28

4. The Leasehold Improvements account includes (a) the P450,000 cost of improvements with a total estimated useful life of 12 years, which Acero as tenant made to leased premises in January 2012, and movable assembly line equipment costing P450,000 that was installed in the leased premises in December 2013. Acero paid its rent in full during 2013. A 10-year non-renewable lease was signed January 3, 2012, for the leased building that Acero used in manufacturing operations. 21. What is the carrying value of Patents on December 31, 2013? A. P3,846,000 C. P3,653,700 B. P2,664,750 D. P2,805,000 22. What amount of impairment loss should be recognized in 2012? A. P 0 C. P1,080,000 B. P1,710,000 D. P990,00 23. What is the carrying value of Licensing Agreement no.2 on December 31,2013? A. P1,800,000 C. P1,728,000 B. P1,920,000 D. P1,620,000 24. What is the carrying value of Licensing Improvements on December 31, 2013? A. P375,000 C. P360,000 B. P405,000 D. P720,000 25. What is the adjusted balance of the Machinery and Equipment account on December 31, 20-13? A. P3,570,000 C. P4,275,000 B. P5,061,000 D. P4,611,000 PROBLEM NO.6 Presented below are two (2) independent situations. Answer the questions at the end of each situation. On December 31, 2012, BANTAY COMPANY acquired a piece of equipment from Sendong Company by issuing a P600, 000, note payable in full on December 1, 216. Bantay’s credit rating permits it to borrow funds from its several lines of credit at 10%. The equipment is expected to have a 5-year life and a P75,000 salvage value. 26. What is the equipment’s carrying value on December 31, 2015? A. P208,922 C. P298,749 B. P163,922 D. P163,922 27. What is the note payable’s carrying value December 31, 2015? A. P545,453 C. P298,749 B. P465,866 D. P163,922 ONDONG COMPANY purchased machinery on December 31, 2012, paying P120,000 down and agreeing to pay the balance in four equal instalments of P90,000 payable each December 31. Implicit in the purchase price is an assumed interest of 12%. 28. What is the cost of the machinery purchased on December 31 2012? A. P273,362 C. P480,000 B. P393,362 D. P349,624 29. How much interest expense should reported in Ondong’s Income statement for the year ended December 31, 2014? A. P18,253 C. P25,940 B. P32,803 D. P64,060 30. What is the carrying value of the note at December 31,2015? A. P152,105 C. P180,000 B. P216,165 D. P80,358 PROBLEM NO.7 LYKA COMPANY’s net income for the past three years are presented below; 2013 2012 2011 P480,000 P450,000 P360,000 During the 2013 year-end audit, the following items come to your attention: 1. Lyka bought a truck on January 1, 2010 for P196,000 with a P16,000 estimated salvage value and a six-year life. The company debited an expense account and credited cash on the purchase date for the entire cost of the asset. (Straight-line method) A1 PASSERS TRAINING, RESEARCH, REVIEW AND DEVELOPMENT COMPANY// 29

2. During 2013, Lyka changed from the straight-line method of depreciating its cement plant to the double-declining balance method. The following computations present depreciation on both bases:

Straight-line Double-declining

2013 36,000 46,080

2012 36,000 57,600

2011 36,000 72,000

The net income for 2013 was computed using the double-declining balance method, on the January 1, 2013 book value over the useful life remaining at the time. The depreciation recorded in 2013 was P72,000. 3. Lyka, in reviewing its provision for uncollectible during 2013, has determined that 1% is the appropriate amount of bad debt expense to be charged to operations. The company had use 1/2 of 1% as its rate in 2012 when the expense had been P18,000 and P12,000, respectively. The company recorded bad debt expense under the new rate for 2013. The company would have recorded P6,000 less of bad debt expense on December 31, 2013 under the old rate. 31. What is the entry necessary to correct the books for the transaction in part 1 of the problem, assuming that the books for the current year are still open? A. Equipment 196,000 Depreciation expense 30,000 Accumulated depreciation 120,000 Retained earnings 106,000 B. Equipment 196,000 Accumulated depreciation 90,000 Retained earnings 106,000 C. Equipment 166,000 Accumulated depreciation 30,000 Retained earnings 136,000 D. Equipment 196,000 Depreciation expense 30,000 Accumulated depreciation 60,000 Retained earnings 166,000 32. What is the correct net income to be reported in 2011? A. P330,000 C. P360,000 B. P336,000 D. P294,000 33. What is the correct net income to be reported in 2012? A. P398,400 D. P480,000 B. P420,000 C. P441,600 34. What is the correct net income to be reported in 2013? A. P510,000 C. P444,000 B. P428,400 D. P450,000 35. Which of the following is (are) the proper time period(s) to record the effects of a change in accounting estimate? A. Current period and prospectively C. Retrospectively only B. Current period and retrospectively D. Current period only PROBLEM NO.8 At the beginning of 2013, an entity grants 50 share option each to 500 employees. The grants is conditional upon the employees remaining in the entity’s employ during a vesting period of three years. The exercise price at grant date is estimated at P60. However, the exercise price drops to P40 if the entity’s earning increase by at least an average of 10% per year over the three-year period. On grant date, the entity estimates that the fair value of the share options, with an exercise price of P40, is P20 per option. If the exercise price is P60, the entity estimates that the share options have a fair value of P18 per option.

A1 PASSERS TRAINING, RESEARCH, REVIEW AND DEVELOPMENT COMPANY// 30

The following actual events occurred: 2013  

2014   2015  

30 employees have left. The entity expects on the basis of a weighted average probability that a further 30 employees will leave during 2014 and 2015, respectively. The entity’s earning increase by 12% and the entity expects that earning will continue to increase at this rate over the next two years. The entity therefore expects that the earnings target will be achieved, and hence the share options will have an exercise price of P40. At year end, a further 35 employees have resigned. The entity expects that a further 30 employees will leave during 2015. The entity’s earning increased by 13% and it continues to expect that the earning target will be achieved. A further 28 employees have left by the end the year. Due to a general decrease in market demand, the entity’s earnings increased by only 3% because the earnings target was not achieved , the 50 vested share options for each employee have exercise price of P60.

Base on the preceding information, determine the following: 36. Compensation expense for 2013 A. P315,000 B. P96,300 37. Compensation expense for 2014 A. P96,300 B. P133,333 38. Compensation expense for 2015 A. P16,667 B. P135,000 39. Share options outstanding at the end of 2014 A. P270,000 B. P405,000 40. Share options outstanding at the end of 2015 A. P405,000 B. P0

C. P136,667 D. P122,100 C. P136,667 D. 135,000 C. P133,333 D. P96,300 C. P133,333 D. P229,633 C. P410,000 D. P366,000

PROBLEM NO.9 In making the first audit of the Delivery Equipment account of DELTA CORPORATION as of December 31, 2013, you encounter the encounter the following facts.

9/1/12

DELIVERY EQUIPMENT Particulars Trucks 1,2,3, and 4 Replacement of truck 3 tires Truck 5 Reconditioning of truck 4, which was damaged in a collision Insurance recovery on Truck 4 accident

10/1/12 4/1/13 5/2/13 6/30/13 12/1/13

Sale of Truck 2 Truck 6 Repainting of Truck 4 Truck 7 Cash received on lease of Truck 7

Date 1/1/11 3/15/12 7/1/12 7/10/12

Debit P3,200,000 25,000 800,000

Credit

35,000 P 33,000 1,000,000 27,000 720,000

600,000 150,000

22,000

Accumulated Depreciation Date 12/31/13 12/31/12 12/31/13

Particulars Depreciation expense Depreciation expense Depreciation expense

Debit

Credit P300,000 300,000 300,000

A1 PASSERS TRAINING, RESEARCH, REVIEW AND DEVELOPMENT COMPANY// 31

Additional Information: 1. Only July 1, 2012 Truck 3 was traded in for a new truck, Truck 5, costing P850,000; the selling party allowed a P50,000 trade in value for the old truck. 2. On April 1,2013, Truck 6 was purchased for P1,000,000; Truck 1n and cash of P1,000,000; Truck 1 and cash of P850,000 being given for the new truck. 3. You are instructed by the senior in-charge of the audit to accept the depreciation rate of 20% by unit basis. 4. Unit cost of Truck 1 to 4 is at P800, 000 each. 41. What is the correct cost of Truck No. 5? A. P850,000 C. P900,000 B. P800,000 D. P560,000 42. What is the correct depreciation expense for 2012? A. P725,000 C. P605,000 B. P305,000 D. P600,000 43. What is the correct depreciation expense for 2013? A. P552,000 C. P712,000 B. P592,000 D. P300,000 44. The entry to correct the depreciation charges for the years 2011 through 2013 should include a credit to Accumulated Depreciation of A. P645,000 C. P900,000 B. P937,000 D. P292,000 45. The balance of the Delivery Equipment account at December 31, 2013 should be A. P5,770,000 C. P4,170,000 B. P3,320,000 D. P3,370,000 PROBLEM NO.10 SUMPAAN CORP. began operations in 2008. On July 15,2013, a fire broke out in the company’s warehouse destroying all inventory and many accounting records. The following information was assembled from the microfilmed records. All sales and purchases are on account. Jan. 1, 2013 Inventory P 287,700 Accounts receivable 261,180 Accounts Payable 176,280 Collections from customers, Jan. 1,2013---July 15,2013 Payments to suppliers, Jan. 1,2013-July 15,2013 Goods out on consignment on July 15,2013, at cost Goods in transit at July 15,2013, purchased FOB shipping point (included in the July 15 accounts payable balance)

July 15,2013 P257,780 245,700 1,507,600 975,000 97,500 34,750

The following is a summary of prior year’s sales and gross profit on sales: 2010 2011 2012 Sales P 1,252,000 P 1,410,000 P 1,360,000 Gross 375,600 366,600 462,400 46. What is the company’s average gross profit ratio based on its prior years’ sales? A. 26% C. 30% B. 34% D. 29% 47. What is the company’s total sales for the period January 1 through July 15 of the current year? A. P1,504,200 C. P1,360,380 B. P1,511,000 D. P1,009,670 48. What is the company’s total purchases for period January 1 through July 15 of the current year? A. P905,580 C. P1,044,420 B. P912,170 D. P1,009,670 49. What is the company’s estimated inventory on July 15,2013, before the fire? A. P186,605 C. P146,930 B. P244,430 D. P279,180 50. What is the inventory fire loss? A. P146,930 C. P132,250 B. P186,605 D. P112,180 A1 PASSERS TRAINING, RESEARCH, REVIEW AND DEVELOPMENT COMPANY// 32

PROBLEM NO.11 YANG CO. Started operations on October 1,2008. Its accounts at June 30, 2011 included the following balances: Machinery (at cost) Accumulated depreciation-machinery Vehicles (at cost; purchased February 20, 2009) Accumulated depreciation-vehicles Land (at cost; purchased March 20,2011) Building (at cost; purchased March 20,2011) Accumulated depreciation-building Land improvements (at cost; purchased March 20,2011) Accumulated depreciation-land improvements

P 196,000 95,772 320,000 178,880 150,000 581,200 6,840 36,000 600

Details of machines owned at June 30,2011 were: Machine 1 2 3

Purchase Date October 2,2008 December 27,2008 July 29,2009

Cost P50,000 84,000 62,000

Useful Life 4 years 5 years 4 years

Residual Value P5,000 8,000 6,000

Additional Information: a. Yang calculates depreciation to the nearest month and balances the records at month-end. Recorded amounts are rounded to the nearest peso, and the reporting date is June 30. b. Yang uses straight-line depreciation for all depreciable assets except vehicles, which are depreciated on the diminishing balance at 30% p.a. c. The vehicles account balance reflects the total paid for four identical delivery vehicle, which cost P80,000 each. d. On acquiring the land and building, Yang estimated the building’s useful life and residual value at 20 years and P34,000 respectively. e.

The land improvements account balance reflects a payment of P36,000 made on march 20, 2010 for driveways and a car park. On acquiring these land improvements, Yang estimated their useful life at 15 years with no residual value.

The following transactions occurred from July 1,2011: Aug. 03, 2011 Purchased a new machine (machine 4) for a cash price of P72,000. Installation costs of P3,600 were also paid. Yang estimated the useful life and residual value at five years and P7,000 respectively. Nov. 15, 2011 Paid vehicle repairs of P1,200. Dec. 30, 2011 Exchanged one of the vehicles for items of fixtures that had a fair value of P34,000 at the date of exchange. The fair value of the vehicle at the date of exchange was P32,000. The fixtures originally cost P100,000 and had been depreciated by P62,000 to the date of exchange in the previous owner’s books. Yang estimated the fixtures’ useful life and residual value at five years and P5,000 respectively. Mar. 10, 2012 Sold machine 1 for P10,000 each. June 30, 2012 Recorded depreciation expense. Sep. 20, 2012 Traded in machine 3 for a new machine (machine 5). A trade-in-allowance of P20,000 was received for machine 3 and P68,000 was paid in cash. Yang estimated machine 5’s useful life and residual value at six years and P10,000 respectively. Dec. 30, 2012 Scrapped machine 2, as it was surplus to requirements and no buyer could be found for it. A1 PASSERS TRAINING, RESEARCH, REVIEW AND DEVELOPMENT COMPANY// 33

Feb. 08, 2013 Paid P16,000 to overhaul machine 4, after which machine 4’s useful life was estimated at two remaining years and its residual value was revised to P10,000. June 30, 2013 Recorded depreciation expense. Based on the preceding information, determine the following: 51. Depreciation expense on machinery for the year ended June 30, 2012. A. P53,027 C. P49,277 B. P50,420 D. P41,777 52. Gain on exchange on December 30, 2011. A. P4,012 C. P4,000 B. P2,012 D. P 0 53. Total depreciation expense on all depreciable assets for the year ended June 30,2012. A. P125,216 C. P113,489 B. P111,281 D. P118,781 54. Gain on trade in of machine no. 3 on September 20, 2012. A. P23,667 C. P7,667 B. P2,333 D. P 0 E. 55. Total depreciation expense on all depreciable assets for the year ended June 30, 2013. A. P98,951 C. P106,551 B. P109,451 D. P82,951 PROBLEM NO.12 SAM, INC grants its customers 30 days credit. The company uses the allowance method for its uncollectible accounts receivable. During the year, a monthly bad debt accrual is made by multiplying 2% times the amount of credit sales for the month. At the fiscal year-end of uncollectible accounts is adjusted accordingly. At the end of 2012, accounts receivable were P3,750,000 and the allowance account had a credit balance of P318,000. Accounts receivable activity for 2013 was as follows: Credit sales P11,400,000 Write-offs 246,000 Collections ? The company’s controller prepared the following aging summary of year-end accounts receivable: Age Group Amount Percent Uncollectible 0-60 days P2,475,000 2% 61-90 days 660,000 10% 91-120 days 150,000 30% Over 120 days 384,000 40% Total P3,669,000 Based on the preceding information, determine the following: 56. Allowance for uncollectible accounts before year-end adjustment A. P314,100 C. P300,000 B. P318,000 D. P228,000 57. Required balance in the allowance for uncollectible accounts at December 31,2013 A. P314,100 C. P318,000 B. P300,000 D. P228,000 58. Correct bad debt expense for 2013 A. P213,900 C. P14,100 B. P242,100 D. P3,369,0 E. 00 59. Net realizable value of accounts receivable at December 31, 2013 A. P3,426,900 C. P3,354,900 B. P3,669,000 D. P3,369,000 60. Collections from customers during 2013 A. P14,823,000 C. P14,904,000 B. P11,235,000 D. P3,996,000 A1 PASSERS TRAINING, RESEARCH, REVIEW AND DEVELOPMENT COMPANY// 34



A1 PASSERS TRAINING, RESEARCH, REVIEW & DEVELOPMENT COMPANY 2nd Floor Sommerset Bldg., Lopez Jaena St. Jaro, Iloilo City Tel. No.: (033) 320-2728; 09106547262 Email Address: [email protected] BOARD OF CERTIFIED PUBLIC ACCOUNTANT CERTIFIED PUBLIC ACCOUNTANT Licensure Examination SET B

AUDITING PROBLEMS GENERAL INSTRUCTIONS: 1. This test booklet contains 45 test questions. 2. Read INSTRUCTIONS TO EXAMINEES printed on your answer sheet. 3. Shade only one (1) box for each question on your answer sheets. Two or more boxes shaded will invalidate your answer. 4. AVOID ERASURES. INSTRUCTIONS: 1. Detach one (1) answer sheet from the bottom of your Examinee ID/Answer Sheet Set. 2. Write the subject title “AUDITING PROBLEMS” on the box provided. 3. Shade Set Box “A” on your answer sheet if your test booklet is Set A; Set Box “B” if your test booklet is Set B. __________________________________________________________________________________________

PROBLEM 1- ANYANG CORP. 1. D Unrealized loss- OCI, 2013 (P13,635,000 CV-P9,390,000 FV) 2. D Unrealized gain, Jan. 1, 2013 (P13,005,000 – P12,330,000) Unrealized gain related to securities sold (P4,830,000 – P4,550,000) Unrealized loss in 2013 Cumulative unrealized loss, Dec. 31, 2013 3. B Nut realized gain in 2014 Loss on sale of Kongga shares Gain on sale of Kandong securities Fair value of Kandong securities sold Proceeds from sale 4. B At fair values 5. B Unrealized loss, Dec. 31, 2013 Unrealized gain on Kandong securities sold (P5,850,000 – P5,250,000) Unrealized loss on Kongga shares sold (P780,000 – P720,000) Adjusted balance Unrealized gain in 2014 (P13,140,000 – P2,820,000) Unrealized gain, Dec. 31, 2014

P4,245,000 675,000 (330,000) (4,245,000) (P3,900,000) P1,440,000 270,000 1,710,000 5,850,000 P7,560,000 P3,900,000 600,000 (60,000) 4,440,000 (10,320,000) P5,880,000

PROBLEM 2 – DARWIN CORP. 6. C Land (appraised value) 7. A Total purchase price Land appraise Balance for allocation Building (9,450/13,500 x P9,000,000) Renovations

P26,100,000 P36,000,000 (26,100,000) P 9,900,000 P6,930,000 9,000,000

A1 PASSERS TRAINING, RESEARCH, REVIEW AND DEVELOPMENT COMPANY// 35

Capitalized Interest(9,000,000 x 12%) Total cost of building

1,060,000 P17,010,000

8. D Machinery (4,050/13,500 x P9,900,000) P2,970,000 9. D Cost of building 2013 depreciation (P17,010,000 x 10% x ½) 2014 depreciation (P17,010,000 – P850,500 = P16,159,500 x 10%) Net book value, Dec. 31, 2014 *150% x 1/15 10. D Cost of machinery 2013 depreciation (P2,970,000 x 40%* x ½) 2014 depreciation (P2,970,000 – 594,000) P2,376,000 x 40%) Net book value, Dec. 31, 2014  200% x 1/5

P17,010,000 (850,500) 1,615,950 P14,543,550

P2,970,000 (594,000) (950,400) P1,425,600

PROBLEM 3 – KANDARA CO/BULDOG CO/TANOGO CO. 11. B Issuance of ordinary shares (P15 x 240,000) Retained earnings (735,000 – P112,250) Shareholders’ equity, Dec. 31, 2012 2013 Jan. 10 issuance of ordinary shares (P16.50 x 35,000) Apr. 2 issuance of pref. Shares (F7x 80,000) Oct. 25 issuance of pre3f. Shares(P10 x 20,000) Net Income

P3,600,000 622,750 4,222,750

577,500 560,000 200,000 732,000

Dividends declared (420,000) Shareholders’ equity, Dec, 31, 2013 12. D Retained earnings, Jan. 1, 2013 Net income Dividends Appointment for the purchase of machinery Unappropriated retained earnings Dec..31, 2013 13. B a.) P30 x 60,000 b.) P35,000 – (P2 x 1,000) c.) P33,000 – (P2 x 1,500) d.) P885,000 + P240,000 = P1,125,000 – (P2 x 137,500) e.) PP35 x 29,500 Total share premium 14. A Jan. 1 Shares outstanding Feb. 15 Issuance May 12 Bond conversion (50 x 1,000) 50,000 June 11 Share dividend (580,000 x 10%) June 30 Shares outstanding Dividend rate Dividends to be paid 15. D 1st quarter (500,000 + 30,000 = 530,000 x P2) 2nd – 4th quarter (P1,276,000 x 3) Total dividends to be paid

P5,872,250 P622,750 732,000 (420,000) (147,500) P787,250 P1,800,000 33,000 30,000 850,000 1,062,000 P3,775,000 500,000 30,000

58,000 638,000 P2 P1,278,000 P1,060,000 3,828,000 P4,888,000

A1 PASSERS TRAINING, RESEARCH, REVIEW AND DEVELOPMENT COMPANY// 36

PROBLEM 4- LAROSA CORPORATION 16. A

Operating Income for 2012, as reported Income from discontinued operations Income from continuing operations before tax Income tax (30%) Income from continuing operations for 2013 17. C Operating Income for 2013, as reported Loss from discontinued operations Income from continuing operations before tax Income tax (30%) Income from continuing operations for 2013 18. D Gain on sale of assets (P300,000 x 70%) Loss from discontinued operations (170,625 x 70%) Post-tax Income from discontinued operations 19. D Post-tax loss from discontinued operations (P170,625 x 70%) 20. D Post-tax loss from discontinued operations (P170,625 + P450,000*= P620.625 x 70%) Implement loss = P1,875,000 FV- P2,325,000 CV

P1,425,000 (121,875) 1,303,125 390,938 P912,187 P1,462,500 170,625 1,633,126 489,938 P1,143,187 P210,000 (119,438) P90,562 P119,438 P434,438

PROBLEM 5 – ACERO CORP. 21. B Cost of patents Amortization for 2013 (P2,805,000/20 years) Carrying value, Dec. 31, 2013 22. C Cost of licensing agreement no.1 Recoverable value Impairment loss 23. D Cost of licensing agreement no.2 (P1,620,000 + P180,000) Amortization (P1,800,000/10) Carrying value, Dec. 31, 2013 24. C Cost of leasehold Improvements Accumulated depreciation (450,000 x 2/10) Carrying value, Dec. 31, 2013 25. B Machinery and equipment per books, 12/31/13 Cost of Improving machinery Assembly line equipment Adjusted balance, Dec. 31, 2013 PROBLEM 6 – BANTAY COMPANY/ONDONG COMPANY 26. A Cost of equipment (P600,000 x 0.68301) Accumulated depreciation, Dec. 31, 2015 (P409,806 – P75,000 = P334,806 x 3/5) Carrying value, Dec. 31, 2015 27. A Note payable, Dec. 31, 2015 28. B Down payment PV of instalment payments (P90,000 x 3.03735) Cost of machinery 29. C Interest expense for 2015 30. D Note payable, Dec. 31, 2015

P2,805,000 (140,250) P2,664,750 P1,800,000 720,000 P1,080,000 P1,800,000 (180,000) P1,620,000 P450,000 (90,000) P360,000 P3,570,000 1,041,000 450,000 P5,061,000

P409,806

(200,884) P209,922 P545,453 P120,000 273,362 P393,362 P25,940 P80,358

PROMBLEM 7 – LYKA COMPANY 31. A 32. A 2011 (P360,000 – P30,000) P330,000 33. B 2012 (450,000 – P30,000) P420,000 34. D 2013 (P480,000 – P30,000) P450,000 35. A A1 PASSERS TRAINING, RESEARCH, REVIEW AND DEVELOPMENT COMPANY// 37

PROBLEM 8 – LYKA COMPANY

Year 1 2 3

Calculation P20 x 410 x 50 x 1/3 P20 x 405 x 50 x 2/3 P18 x 407 x 50

Compensation Expense for Period P136,667 133,333 96,300

Cumulative Compensation Expense P136,667 270,000 366,300

PROBLEM 9 – DELTA CORPORATION

1 (Traded in 4/1/13) 2 (Sold 10/1/12) 3 (Traded in 7/1/12) 4 5 (Acquired 7/1/12) 6 (Acquired 4/1/13) 7 (Acquired 6/30/13)

2011 P160,000 160,000 160,000 160,000 -

Correct depreciation P640,000 Depreciation per client (300,000) Understatement P340,000

2012 P160,000 120,000 80,000 160,000 85,000 -

P605,000 (300,000) P305,000

PROBLEM 10 – SUMPAAN CORP. 36. C 30% + 28% + 34%/3 37. C Accounts receivable July 15 Collections from customers Total Accounts receivable, Jan. 1 Estimated sales, Jan. 1 – July 15 38. C Accounts payable, July 15 Payments to suppliers Total Accounts payable, Jan. 1 Estimated purchases, Jan. 1 – July 15 39. D Inventory, January 1 Estimated purchase Goods available for sale Estimated cost of goods sold(P1,504,200 x 70%) Estimated inventory, July 15 40. A Estimated inventory, July 15 Goods out on consignment Goods in transit Inventory fire loss PROBLEM 11 – YANG CO. 41. M1 (P50,000 - P5,000)/4 x 8/12 M2 (P64,000 - P8,000)/5 M3 (P62,000 – P6,000)/4 M4 (P75,600 –P7,000)/5 x 11/12 Total depreciation expense – machinery (C) 42. Cost of vehicle trade in Accumulated depreciation, Dec. 30, 2011: Balance, June 30, 2011 (P178,880 x ½ ) P44,720 Depreciation, 7/1/11 – 12/30/11 (P320,000 – P178,880 = P141,120 x ¼ x 30% x 6/12) 5,292 Carrying value, Dec. 30, 2011 Fair value of vehicle traded in Gain on exchange, Dec. 30, 2011 (B)

2013 P40,000 160,000 170,000 50,000 72,000

P592,000 (300,000) P292,000

Total P360,000 280,000 240,000 480,000 225,000 150,000 72,000

P1,837,000 (900,000) P937,000 30% P257,780 1,507,600 1,765,380 (261,180) P1,504,200 P245,700 975,000 1,220,700 (176,280) P1,044,420 P287,700 1,044,420 1,332,120 (1,052,940) P279,180 P279,180 (97,500) (34,750) P146,930

P7,500 15,200 14,000 12,577 P49,277 P80,000

50,012 29,988 32,000 P2,012

A1 PASSERS TRAINING, RESEARCH, REVIEW AND DEVELOPMENT COMPANY// 38

43. Machinery Vehicle (P141,120 x ¾ x 30%) + P5,292

49,277

Building (P581,200 – P34,000 = P547,200/20) Land Improvements (36,000/15) Fixtures (32,000 – P5,000 = P27,000/5 x 6/12) Total depreciation expense (D) 44. Cost of machine no.3 Accumulated depreciation, Sep. 20, 2012 (P62,000 – P6,000 = P56,000/4 x 38/12) Carrying value Trade in allowance Gain on trade in (B) 45. Machinery: M2 (P84,000 – P8,000 = P76,000/5 x 6/12) M3 (P62,000 – P6,000 = P56,000/4 x 3/12) M4 – July 1, 2012 – Feb. 8, 2013: (P75,600 – P7,000 = P66,600/5 x 7/12) - Feb. 8, 2013 – June 30, 2013 (P61,020/2 years x 5/12) M5 (P88,000 – P10,000 = P78,000/6 x 9/12) Vehicles Cost (P320,00 x ¾ ) Accum. Depr., June 30, 2012 (P178,880 – P50,012 + P37,044 Carrying value, June 30, 2012 Multiply by depreciation rate Building (P581,200 – P34,000 = P547,200/20) Land improvements P36,000/15) Fixtures (P32,00 – P5,000 = P27,000/5) Total depreciation expense, June 30, 2013 (A)

37,044 27,360 2,400 2,700 P118,781

17,667

P7,600 3,500 P8,003 12,712 9,750

20,175 P41,565

P240,000 (165,912) 74,088 30%

22,226 27,360 2,400 5,400 P98,951

PROBLEM 12 – SAM,INC. 46. C

57. A

58.B

59.C

60.B

A1 PASSERS TRAINING, RESEARCH, REVIEW AND DEVELOPMENT COMPANY// 39



A1 PASSERS TRAINING, RESEARCH, REVIEW & DEVELOPMENT COMPANY 2nd Floor Sommerset Bldg., Lopez Jaena St. Jaro, Iloilo City Tel. No.: (033) 320-2728; 09106547262 Email Address: [email protected] BOARD OF CERTIFIED PUBLIC ACCOUNTANT CERTIFIED PUBLIC ACCOUNTANT Licensure Examination SET A

BUSINESS LAW AND TAXATION GENERAL INSTRUCTIONS: 1. This test booklet contains 70 test questions. 2. Read INSTRUCTIONS TO EXAMINEES printed on your answer sheet. 3. Shade only one (1) box for each question on your answer sheets. Two or more boxes shaded will invalidate your answer. 4. AVOID ERASURES. INSTRUCTIONS: 1. Detach one (1) answer sheet from the bottom of your Examinee ID/Answer Sheet Set. 2. Write the subject title “BUSINESS LAW AND TAXATION” on the box provided. 3. Shade Set Box “A” on your answer sheet if your test booklet is Set A; Set Box “B” if your test booklet is Set B. __________________________________________________________________________________________

1. If debtor gives a property to his creditor who accepts it in payment of an obligation in money, this will be governed by a. Cession c. Exchange b. Barter d. Sales 2. D owes C P6,000. No date for payment was stipulated by the parties. Which is correct? a. C can require D to pay when the period arrives b. C can require D to pay at anytime c. D is not liable to C because the obligation is void there being no date of payment d. D is not required to pay unless C goes to court to require D to pay 3. D borrowed P100,000 from C. the obligation is secured by a mortgage on D’s land and building C registered the mortgage with the Register of Deeps. Thereafter, D sold the land and building to B who was not aware of the existence of the mortgage at the time of sale since only the photocopy of the transfer certificate of title which did not yet contain the annotation of the mortgage as shown to him. It was only when he went to the Register of Deeps to register the sale of the land and building to him that the learned of the mortgage. Which is correct? a. C can collect from D and if D cannot pay, C can foreclose the mortgage b. C can collect from D cannot pay, C cannot foreclose the mortgage c. C cannot collect from D. he can only go after the mortgage which was given as security d. C cannot foreclose the mortgage because B is a buyer in good faith 4. If the illegal contract between the parties is a criminal offense but only one party is guilty, which of the following is wrong? a. The guilty party will be criminally prosecuted b. Neither party may compel the other to comply with his undertaking c. The instruments shall be confiscated in favour of the government d. The innocent party shall have no right to receive what he had given 5. A, a bachelor who has sired many children by different women, donated a house and lot to T, his illegitimate son. The deep of donation and its acceptance were in a public instrument and a new certificate of title to the house and lot was issued in the same of T. Later, A discovered that T was not his son. Can A annul the contract? a. No, because all requisites have been complied with and a new title had been issued in the name of T b. Yes, A may annul the contract on the ground of mistake as to the identity of T c. No, but A can ask for rescission because he suffered damages d. Yes, the contract is void ab inition 6. In January 2002, S, 17 years old, sold his only car to B, 21 years old. The sale was without the knowledge of G, the guardian of S. Assuming that an annulment case is field today, which is correct? a. S may bring the action for annulment b. G may bring the action for annulment c. B may bring the action for annulment d. Annulment will not prosper whoever will file it A1 PASSERS TRAINING, RESEARCH, REVIEW AND DEVELOPMENT COMPANY// 40

7. Which of the following statements is true as regards a void contract? a. A void contract may be ratified by the acceptance by a party to the contract of a benefit under its terms b. The right to set up the defends of illegality of a void contract maybe waived if the illegality is not raised within a reasonable time c. If a void contract is notated by a valid one such notation is valid d. The right to set up the defence of illegality of a void contract is imprescriptible 8. It is contract by virtue of the terms of which the parties thereto promise and obligate themselves to enter into another contract at a further time, upon happening of certain events, or the fulfilment of certain conditions a. Resolutory contract c. Suspensive contract b. Option contract d. Auto-contact 9. Which of the following is a feature common to chattel mortgage and pledge? a. Deficiency is recoverable in case of sale of the thing pledge/ mortgage b. The object of the contract is a movable property c. The excess of the proceeds of sale over the amount of the obligation of the belongs to the pledgor/mortgagor d. An affidavit of good faith is required to bind third persons 10. P appointed A as commission agent to sell’s goods for P10,000 cash. A however sold credit for P11,000 without P’s consent. Based on the foregoing facts, which of the following is correct? a. P may demand immediate payment of P11,000 cash b. P may demand immediate payment of P10,000 cash and collect P1,000 later c. P may ratify the sale on credit for P11,000 and immediately demand P10,000 from A d. A is entitled to the excess of P1,000 is P did not ratify the sale on credit 11. F made a telephone call to his son S, for S to sell F’s land. In the deep of absolute sale which was acknowledged before a notary public, S signed as agent of F. Later, in the presence of two witnesses, F ratified the sale executed by S. Which is correct? a. The sale was void at the start but was validated upon the ratification by F b. The sale remained void despite the ratification by F c. The sale was valid from the start since it was made in a public instrument and F, the owner ratified the sale d. The sale was merely unenforceable at the start but the ratification validated the contract 12. A bought a residential house and lot from B Realty for P2M giving a down payment of P200,000 and promising to pay the balance of P1.8M in 15 years in monthly instalments of P10,000. After paying 72 instalments A defaulted in the payment of subsequent instalments. Despite the grace period given, he was not able to make any further payments. Accordingly, B Reality cancelled the sale. How much cash surrender value is a entitled to receive? a. P552,000 c. P462,000 b. P396,000 d. P506,000 13. A sum of money paid, or a thing delivered upon the making of a contract for the sale of goods to bind the offer, the delivery and acceptance of which makes the final assent of both parties to the contract a. Option money c. Discount b. Arms d. Deposit 14. A promised to give B P1, 000 if B will hear mass for ten consecutive Sundays. This is a a. Civil obligation c. Moral obligation b. Natural obligation d. Religious obligation 15. A promissory note reads “I promise to pay B P100,000 on Oct. 30, 2012. Sgd. A “. B transferred the note to C. later, X note the promissory note and transferred the note to Y who received the note in good faith. On maturity date, Y presented the note to A and demanded payment. A paid the note in good faith. In this case a. C can recover from A c. A’s obligation is extinguished b. C can recover from B d. C can recover only from either X or Y 16. A sells to B a Sony colored TV for P12,000 payable in twelve equal monthly instalments beginning May 5, 2012 and every 5th day of each month thereafter. The contract provides that upon failure to pay any instalment due, the whole balance becomes due and demandable. To secure the obligation, a chattel for the payment of the TV set was executed. When B defaulted on the seventh and eight instalments, A sued B for the payment of the whole balance of P6,000. The TV set was levied and subsequently sold at a public sale for P5,000. Can A still recover the deficiency of P1,000 from B? a. Yes, if there was stipulation to that effect b. Yes, even though there was no stipulation c. No, any stipulation allowing recovery is void d. No, if there was no stipulation to that effect

A1 PASSERS TRAINING, RESEARCH, REVIEW AND DEVELOPMENT COMPANY// 41

17. A pledged his ring to B for P20,000. A failed to pay his obligation. B sold it at a public auction for P18,000. Can B recover the deficiency? a. Yes, even without stipulation c. No, even if there is stipulation b. Yes, if there is stipulation d. No, unless there is stipulation 18. Using the above data, if the sale is P22,000. Can A recover the excess? a. Yes, even without stipulation c. No, even if there is stipulation b. Yes, if there is stipulation d. No, if there is stipulation 19. C mortgaged his car to B for P200,000. C failed to pay his obligation. B sold it at a public action for P180,000. Can B recover the deficiency? a. Yes, even without stipulation c. No, even if there is stipulation b. Yes, only if there is stipulation d. No, if there is stipulation 20. Using the above data, if the sale is for P220,000, can C recover the excess? a. Yes, even without stipulation c. No, even if there is stipulation b. Yes, only if there is stipulation d. No, unless there is stipulation 21. Essential elements of pledge, except a. Pledgor is the absolute owner of the thing pledged b. The person constituting the pledge has the free disposal of his property or if he is an agent or representative, that he is legally authorized c. The thing pledge must be placed in the possession of the creditor or of a 3rd person by common agreement d. To bind third persons there must be an affidavit of good faith 22. Essential elements of chattel mortgage, except a. The object is a personal or movable property b. The thing mortgage is not delivered to the creditor c. The mortgagor cannot sell without the consent of the mortgage d. Description of the thing mortgage must appear in a public instrument 23. Essential elements of real estate mortgage, except a. In case of foreclosure sale, the mortgage is entitled to the b. The mortgage can sell even without the consent of the mortgage c. The thing mortgaged must be delivered to the creditor d. To bind third persons it must be recorded in the Registry of Property 24. Which of the following statements is not correct? a. If the thing pledged is returned by the pledge to the pledgor, the obligation is extinguished b. In sale at public auction, the pledgor may bid and his bid is valid even if he is the only bidder c. Any stipulation authorizing the pledge to appropriate the thing pledged is void and without effect d. Shares of stock can be the object of pledge or mortgage 25. Which of the following statements is correct? a. Third persons who are not parties to the principal obligation cannot give as security in pledge their property to answer the principal obligation b. An unregistered chattel mortgage is valid upon the parties but void as to innocent third persons c. Pledge is a consensual contract and is perfected from the time the thing pledged is delivered to the creditor d. In mortgage, the mortgage is entitled to the entire proceeds of the sale of the thing mortgaged 26. Which of the following contracts is not void ab initio? a. That whose object is outside the commerce of men b. That whose object did not exist at the time of transaction c. That which contemplates an impossible service d. That which is undertaken in fraud of creditor 27. This contract is not perfected until the delivery of the object of the obligation a. Contract of sale c. Contract of deposit b. Contract to make a pledge d. Contract to make a commodatum 28. As a rule, when a creditor assigns his credit in good faith. A. He warrants the existence of the credit at the time of assignment B. He warrants the legality of the credit at the time of assignment C. He warrants the solvency of the debtor at the time of assignment a. Only A and C c. Only B and C b. Only A and B d. A,B and C 29. An agricultural land is owned by A and D pro-indiviso. D sells his ½ part to R, who is the owner of the adjoining land. When A learned of the sale, he tried to redeem to portion sold by D by reimbursing R with the purchase price and expenses. Which is not correct? a. A can compel R to permit redemption b. A co-owner of a thing may exercise the right of redemption in case the shares of the other co-owners or any one of them are sold to a third person c. The sale to R is valid but A can elect to exercise his right of redemption d. The sale to R is valid and as adjoining land owner hi is the one entitled to redemption A1 PASSERS TRAINING, RESEARCH, REVIEW AND DEVELOPMENT COMPANY// 42

30. A owes B P10,000 with 12% interest. B owes C P1.200. it was agreed between A and B that A would give the interest of P1,200 to C and C communicated his acceptance of the stipulation between A and B. there is a. Stipulation pour atrui c. Compensation b. Pactum commissorium d. Assignment 31. A, B and C are solidary debtors sharing at 1:2:3 of solidary creditors W and Y sharing at 1:2. The obligation is P12,000. If B is a minor and C is insolvent and W condones the obligation of A without the consent of Y, how much can W collect from A? a. P8,000 c. P2,000 b. P6,000 d. P0 32. Using the above information, how much can Y collect from A? a. P4,000 c. P2,000 b. P6,000 d. P0 33. A is obliged to give B either objects No.1 or No.2 or No.3 at A’s option. Before A communicated his choice to B, object No.1 had been destroyed, thru A’s fault and object No.2 had been destroyed by a fortuitous event. B may a. Demand object No.3 only as it is still available b. Demand the price of object No.1 only plus damages because it was destroyed by A’s fault c. Demand the value of object No.1 or object No.3 but without damages d. Demand either object No.3 or the price of object No.1 plus damages 34. Which of the following is a valid stipulation? a. A promises to give B P10,000, and if A fails, A will allow B to have sexual intercourse with her b. A will give B P10,000, if B will kill C c. A agreed to give B a house and lot if B will be his mistress d. A agreed to put poison on the food of B’s wife and if A fails, to pay B P10,000 for damages 35. A,B and C are solidarily indebted to D for P9,000 representing the purchase price of 1 gram of shabu but A’s consent has been obtained by violence and intimidation, and B is a minor. How much can D collect from C? a. P9,000 c. P3,000 b. P6,000 d. P0 36. A cash dividend of P100,000 received by a taxpayer in 2011 from a domestic corporation whose income from Philippine source is 40% of its total income is a. Partly taxable if the is a resident citizen c. Partly taxable if he is a resident alien b. Exempt from final tax it he is a non d. Taxable in full if he is a non-resident resident citizen citizen 37. A cash dividend of P100,000 received by a taxpayer from a foreign corporation whose income from Philippine source is 50% of its total income is a. Exempt from income tax if it is a domestic corporation b. Exempt from income tax if it is a foreign corporation c. Taxable in full if he is a resident citizen d. Taxable in full if he is a resident alien 38. As to scope of legislature power to tax, which of the following is correct? a. The power to tax supreme, plenary, comprehensive and without any limit because the existence of the government is a necessity b. The discretion of Congress in imposing taxes extends to the mode, method and kind of tax, even if the constitution provides otherwise c. Congress has the right to levy a tax of any kind at any amount at it sees fit, even in the absence of any constitutional provision d. The sole arbiter of the purpose for which taxes shall be levied is Congress, provided the purpose is public and the courts may not review the levy of the tax to determine whether or not the purpose is public 39. A resident citizen received a prize of P40,000. Which of the following statements is correct in connection with the imposition of final tax on the prize? a. The first P10,000 is part of taxable income while, the remaining P30,000 is subject to 20%final tax b. The whole amount is part of taxable income c. The whole amount of P40,000 shall be subject to 20% final tax d. The first P10,000 shall be exempt from tax, the remaining P30,000 is subject to 20% final tax 40. Under this system, the amount of income tax withheld by the withholding agent is constituted as full and final payment of the income tax due from the payee on the said income a. Creditable withholding tax b. Final withholding tax c. Global tax system d. Scheduler tax system A1 PASSERS TRAINING, RESEARCH, REVIEW AND DEVELOPMENT COMPANY// 43

41. A domestic corporation has the following data for 2012: Excess MCIT 2011 – P15,000 O1 02 Income net of 1% withholding tax P495,000 P792,000 Deductions P480,000 700,000 How much is the income tax still due and payable in the second quarter? a. P3,000 c. P18,000 b. P4,000 d. (P2,000) 42. Statement1- is a co-ownership taxable? Yes, because although the activities of the co-owners are limited to the preservation of the property they derived income there from Statement2- is the share of a co-owner taxable? No, because each co-owner is taxable individually on his distributive share in the net income of the co-ownership a. Both statements are correct c. Only Statement 1 is correct b. Both statement are wrong d. Only Statement 2 is correct 43. Jun Jon and Company, a general co-partnership has the following data income and expenses for 2012: Gross income P 2,400,000 Expenses 1,000,000 Dividend received from a domestic corporation 90,000 Interest on bank deposit (net) 10,000 Partners Jun and Jon share profits and losses in the ratio of 80% and 20%, respectively .The income tax payable by the partnership is a. P420,000 c. P447,000 b. P450,000 d. P423,000 44. The taxes withheld on the respective share of Jun and Jon in the 2012 partnership income are a. Jun – P78,400 and Jon – P19,600 c. Jun – P129,600 and Jon – P21,600 b. Jun – P117,600 and Jon – P19,600 d. Jun – P86,400 and Jon – P21,600 45. A Marketing incurred the following expenses in relation to its business during the taxable year: Provision for bad debts P 15,000 Research and development cost, treated as differed expense 500,000 Contributions during the year: To the government for priority programs in sport 50,000 To the government for public purposes 10,000 To the Catholic church for charitable purposes 25,000 Additional Information: a. Allowance for doubtful accounts per aging of accounts receivable a beginning and ending balance of P20,000 and P30,000, respectively b. Accumulated depreciation on machine at the beginning and end of the year amounted to 100,000 and P110,000, respectively c. During the year the firm sold a machine with a cost and accumulated depreciation of P300,000 and P30,000, respectively d. The research and development cost was incurred in the preceding year but the benefit was received during the taxable year e. Taxable income before a to d and contributions amounted to P300,000 If A Marketing is a sole proprietorship the allowance before personal exemptions a. P80,000 c. P70,000 b. P85,000 d. P65,000 46. Furthermore the amount of taxable income before personal exemptions a. P220,000 c. P130,000 b. P89,000 d. P115,000 47. Which of the following statements is not correct? a. Compensation income of individuals that do not exceed the statutory minimum wage is exempt from the requirement of withholding tax on compensation b. Compensation income of individuals that do not exceed ten thousand pesos per month is exempt from the requirement of withholding tax on compensation c. Holiday pay, overtime pay, night shift differential pay and hazard pay received by individuals shall be exempt from income tax d. Minimum wage earners who are receiving on the statutory minimum wage are not subject to withholding tax and consequently to income tax

A1 PASSERS TRAINING, RESEARCH, REVIEW AND DEVELOPMENT COMPANY// 44

48. Which of the following statements is correct? a. The monetized value of unutilized vacation leave credit of ten (10) days or less which were paid to the employee during the year are not subject to income tax and to withholding tax b. The salary of an employee on vacation or on sick leave, which are paid not with standing his absence from work, constitutes compensation income, except if not more than 10 days. c. Any amount which is required by law to be deducted by the employee’s compensation and is deemed to be paid to the employee as compensation at the time the deduction is made d. If living quarters or meals are furnished to an employee for the convenience of the employee, the value thereof should be included as part of compensation income 49. On January 1, 2011, A leased his vacant lot for a period of 12 years at p240,000 per year. it was agreed that the lessee will pay the following. a. Rent of P480,000 (for 2011 and 2012) b. Security deposit of P240,000 c. Real property tax at P20,000/year The lease contract provides among others that the lessee will construct a 3-storey building for parking purposes at a cost of P3,600,000 which shall belong to the lessor upon expiration or termination of the lease. The building was completed on July 1, 2011, with an estimated useful life 15 years. A shall report for the year 2011, using the spread-out method a total income from lease of a. P296,543 c. P536,521 b. P573,087 d. P333,087 50. Using the above data the lessee in 2011 can deduct total expenses amounting to a. P656,521 c. P416,521 b. P553,043 d. P573,043 51. Recovery of bad debt written off by a taxpayer: No. 1: P20,000 from accounts written off in a year which had a net income of P200,000 before write-off. (write-off of the year was P20,000); No. 2: P5,000 from written off in a year which had a net loss before write-off of P36,000 (write-off for the year was P5,000); No. 3: P10,000 from written off in a year which had a net income of P8,000 before write-off. (write-off for the year was P12,000); The income from the bad debt recovery is: a. P35,000 c. P26,000 b. P20,000 d. P30,000 52. XYZ, a domestic corporation was determined to be improperly accumulating its earning for the taxable year 2012 based on its records as follows: a. Net income from business P 1,000,000 b. Dividends actually or constructively paid 150,000 c. Income tax paid for the year 270,000 d. Income subjected to 20% final tax (net) 60,000 e. Income exempt from tax 50,000 f. Income excluded from gross income 10,000 g. Net operating loss (2011) 100,000 The tax on improperly accumulated earning is a. P81,500 c. P80,000 b. P71,500 d. P70,000 53. An individual taxpayer, single, has the following data for the current year: Ordinary income P 150,000 Long term capital gain 40,000 Short term capital loss 10,000 How much is the net taxable income a. P100,000 b. P135,000 c. P140,000 d. P110,000

A1 PASSERS TRAINING, RESEARCH, REVIEW AND DEVELOPMENT COMPANY// 45

54. Mabuhay Corporation organized in 2010 has the following data: 2011 2012 Sales P1,700,000 P 2,300,000 Cost 1,700,000 1,425,000 Operating expenses 615,000 480,000 The income tax payable in 2011 is a. P13,000 c. P10,500 b. P5,250 d. P12,250 55. Using the above data, the income tax payable by Mabuhay Corporation in 2012 is a. P118,500 c. P116,000 b. P110,750 d. P105,500 56. A, a resident taxpayer received the following during the taxable year: I. P12,000 cash received has refund of excess insurance premiums paid to an insurance company II. P40,000 worth of goods as prize in “Beer-Gin” talent search III. A computer unit with a fair market value of P48,000 which was awarded to him in recognition of his outstanding achievement in scientific research IV. P25,000 salary per month as an accounting supervisor Which of the above items is (are) exclusion from taxable income subject to graduated rates? a. II only c. I,II and III b. I and II d. I and III 57. Statement 1 – Government owned and controlled corporations are subject to tax unless expressly exempted. Statement 2 - Government agencies performing governmental and proprietary function are exempt from tax unless expressly taxed a. True, true c. True, false b. False, false d. False, true 58. It is aspect of taxation that is administrative in character and the power to exercise it is vested on the Department of Finance. a. Levying c. Imposition b. Collection d. Legislation 59– 60 Mr. C, single, presented to you the following items of income for the taxable year: a. Salary as part time instructor (net of P1,400 w/tax) - P18,600 b. Honorarium as member of the Board of ABC Corp. - P10,000 c. Rental income of apartment house he purchased using his income from salary and professional fees - P12,000 d. Interest income from PNB On deposit maintained under the expanded foreign currency deposit system (net) - $200 On savings deposit (net) - P4,000 e. Compensation income received as a CPA - P25,000 f. Royalty income from book publication - P30,000 g. Prizes received from supermarket raffle - P12,000 h. Winning from PCSO - P100,000 i. Compensation for injuries suffered in an accident - P20,000 j. Bonus and 13th month pay - P10,000 59. The taxable income after exemption of Mr.C is a. P52,000 c. P22,000 b. P17,000 d. P32,000 60. If Mr. C is married, his taxable income after exemption is a. P11,000 c. P4,500 b. P17,000 d. P3,100 61 – 62. A ,single, 49 years old with 2 qualified dependent illegitimate children, presented the following data regarding the items in come he earned during the taxable year: A. Rental income From a commercial land in USA P480,000 From a residential land in Makati P360,000 B. Royalties from books Published in USA P120,000 Published in the Philippine P180,000 C. Interest income on notes receivable The debtor resides in USA P60,000 The debtor resides in the Philippines P72,000 A1 PASSERS TRAINING, RESEARCH, REVIEW AND DEVELOPMENT COMPANY// 46

D. Interest income from Philippines Bank On Peso Currency bank deposit (gross) P80,000 On Foreign Currency bank deposit (gross) P100,000 E. Dividend income from two resident foreign corporation where the gross income from the Philippines for the past three years were equivalent to: 40% of its world income -P60,000 60% of its word income -P40,000 F. Prizes and awards received from: Supermarket raffle From the Philippines – ABC Grocery -P8,000 - DEF Superstore -P10,000 - CHI Mart - P12,000 From USA – KLM Bazaar -P9,000 - NOP Mall P11,000 Ramon Masaysay Award P100,000 Nobel Peace Prize P150,000 G. Prizes and winnings from: PCSO P400,000 USA Lotto P100,000 Mahjong and Pusoy Games P10,000 H. Proceeds of life insurance of his Mother where A was designated as revocable beneficiary P1,000,000 Father where A was designated as irrevocable beneficiary P2,000,000 I. Retirement benefits from his employer after rendering Service for 30 years P1,200,000 J. Net profit from merchandising business From the Philippines P300,000 From USA P200,000 K. Compensation for injuries and damages Actual medical and hospital expenses P100,000 Moral and exemplary P50,000 Lost profits P20,000 L. Sale of shares of stock of a domestic corporation Directly to the buyer, Selling Price P300,000 Cost P120,000 Thru the stock exchange, Selling Price P200,000 Cost P210,000 M. Sale of pieces of jewelry held as capital assets Acquired 10 years ago, Selling Price P140,000 Cost P80,000 Acquired this year, Selling Price P200,000 Cost P210,000 61. If A is a resident citizen, his net taxable income (subject to graduated rates) is a. P2,980,000 c. P2,780,000 b. P1,780,000 d. P2,770,000 62. If A is a non-resident citizen, his net taxable income (subject to graduated rates) is a. P1,924,000 c. P1,724,000 b. P1,940,000 d. P1,914,000 63. Which of the following is subject to 10% final tax? a. P10,000 interest income c. P10,000 raffle prize b. P10,000 royalty from a mining claim d. P10,000 royalty from a song composition 64. The deduction allowed for the payment of premium on health insurance during the taxable year by a resident citizen amounting to P3,000 for the months from August to December is a. P3,000 c. P1,000 b. P2,000 d. P1,250 65. Proceeds of life insurance, if the estate is the beneficiary is a. Part of gross income if revocable b. Part of gross income if irrevocable c. Part of gross income regardless of whether revocable or irrevocable d. Not part of gross income

A1 PASSERS TRAINING, RESEARCH, REVIEW AND DEVELOPMENT COMPANY// 47

66. Which of the following are elements of impact of taxation? I. Levy III. Assessment II. Imposition IV. Collection a. I only c. II and III b. I and II d. III only 67. Which of the aspects of taxation is (are) administrative in nature? I. Levy II. Assessment III. Collection a. I only b. I and II c. II and III d. III only 68. A sold his residential land, a capital asset for P2,000,000. It was acquired in 1990 at a cost of P1,500,000. The fair market value as determined by the BIR is P2,200,000 but the fair market value shown in the schedule of values of City Assessor is P1,950,000. The entire proceeds were utilized for the acquisition of A’s principal residence. The final tax due is a. P120,000 b. P132,000 c. P117,000 d. P0 69. Using the above data, but only P1,600,000 was utilized for the acquisition of A’s principal residence, the final tax due is a. P24,000 b. P26,400 c. P120,000 d. P132,000 70. Acting on the information giving by A, the government seized and confiscated smuggled goods with a fair market value of P1.5M A’s reward will be a. P1.5M which is part of his taxable income b. P1.5M which is subject to 10% final tax c. P1M which is exempt from tax d. P1M which is subject to 10% final tax

A1 PASSERS TRAINING, RESEARCH, REVIEW AND DEVELOPMENT COMPANY// 48



A1 PASSERS TRAINING, RESEARCH, REVIEW & DEVELOPMENT COMPANY 2nd Floor Sommerset Bldg., Lopez Jaena St. Jaro, Iloilo City Tel. No.: (033) 320-2728; 09106547262 Email Address: [email protected] BOARD OF CERTIFIED PUBLIC ACCOUNTANT CERTIFIED PUBLIC ACCOUNTANT Licensure Examination SET A

PRACTICAL ACCOUNTING 1 GENERAL INSTRUCTIONS: 1. This test booklet contains 40 test questions. 2. Read INSTRUCTIONS TO EXAMINEES printed on your answer sheet. 3. Shade only one (1) box for each question on your answer sheets. Two or more boxes shaded will invalidate your answer. 4. AVOID ERASURES. INSTRUCTIONS: 1. Detach one (1) answer sheet from the bottom of your Examinee ID/Answer Sheet Set. 2. Write the subject title “PRACTICAL ACCOUNTING I” on the box provided. 3. Shade Set Box “A” on your answer sheet if your test booklet is Set A; Set Box “B” if your test booklet is Set B. __________________________________________________________________________________________

1. Cocoa Company reported operating expenses in two categories, namely selling and general and administrative. The adjusted trial balance on December 31, 2013 included the following expense accounts: Accounting and legal fees Advertising Freight out Interest expense Loss on sale of long-term investments Officers’ salaries Rent for office space Sale salaries and commissions Research and development expense

140,000 120,000 75,000 60,000 30,000 180,000 180,000 110,000 400,000

One-half of the rented premises is occupied by the sales department. What total amount of the expenses should be included in general and administrative expenses for 2013? a. 410,000 c. 470,000 b. 440,000 d. 810,000 2. Ruthless Company reported the following events and transactions during 2013? Depreciation for 2011 was understated by P300,000. A litigation settlement resulted in a loss of P250,000. The inventory on December 31, 2011 was overstated by P400,000. The entity disposed of its recreational division at a loss of P500,000. The income tax rate is 30%. What is the effect of these events and transactions on 2013 income from continuing operations, net of tax? a. 175,000 c. 525,000 b. 385,000 d. 665,000 3. Klumps Company owned the following investments at year-end before fair value adjustments and amortization: Financial assets at fair value through profit or loss 600,000 Financial assets at fair value through OCI 350,000 Financial assets at amortized cost 470,000 What total amount of noncurrent assets be reported at year-end? a. 1,070,000 b. 950,000 c. 820,000 d. 820,000 or an amount greater or less depending on the circumstances. A1 PASSERS TRAINING, RESEARCH, REVIEW AND DEVELOPMENT COMPANY// 49

4. Wacko Company provided the following data on December 31, 2013: Cash in bank, net5 of bank overdraft of P500,000 Petty cash (unreplenished petty cash expenses, P10,000) Trade notes receivable, including discounted note of P1,000,000 accounted for as a conditional sale. Accounts receivable, net of accounts with credit balances of P1,500,000 Inventory Bond sinking fund Deferred charges Accounts payable, net of accounts with debit balances of P1,000,000 Trade notes payable Bonds payable due on June 30,2014 Accrued expenses

5.

6.

7.

8.

9.

5,000,000 50,000 4,000,000 6,000,000 3,000,000 2,000,000 250,000 7,000,000 4,000,000 2,000,000 500,000

What amount should be reported as total current assets on December 31, 2013? a. 20,040,000 c. 20,050,000 b. 22,040,000 d. 22,290,000 Maricel Company has three manufacturing divisions, each of which has been determined to be a reportable segment. Common costs are appropriately allocated on the basis of each division’s sales in relation to entity’s aggregate sales. In 2013, Division I had sales of P10,000,000, which was 20% of the entity total sales, and has traceable operating costs of P6,000,000. In 2013, the entity incurred costs of P2,000,000 that were not directly traceable to any of the divisions. These costs included general corporate expenses of P500,000. In addition, the entity incurred interest expense of P500,000 which was directly traceable to Division I in 2013. Information about interest expense is regularly provided to the chief operating decision maker. What amount should be reported as operating profit of Division I for 2013? a. 4,000,000 c. 3,600,000 b. 3,700,000 d. 3,200,000 Brod Company has historically reported bad debt expense of 10% of sales in each quarter. For the current year, the entity followed the same procedure in the three quarter of the year. However, in the fourth quarter, the entity determined that bad debt expense for the entire year should be P900,000. Sales were P2,000,000 for first quarter, P1,500,000 for second quarter, P2,500,000 for third quarter, and P4,000,000 for fourth quarter. What amount of bad debt expense should be recognized for the fourth quarter? a. 900,000 c. 400,000 b. 300,000 d. 600,000 Amen Company incurred an inventory loss from market decline of P800,000 on March 31, 2013. The market decline is expected to recover during the year. One June 30, 2013, the net realizable value of the inventory increased by P900,000. What amount of gain should be reported in the quarterly income statement ending June 30, 2013? a. 900,000 c. 100,000 b. 800,000 d. 0 Lure Company purchased equipment for P5,750,000 on January 1, 2012 with a useful life of 10 years and no residual value. On January 1, 2014, the entity classified the equipment as held for sale. The fair value of the equipment on such date is P3,795,000 and the cost of disposal is P115,000. On December 31, 2014, the fair value of the equipment is P4,370,000 and the cost of disposal is P230,000. On December 31, 2014, the entity determined that the criteria as held for sale can no longer be met and decided not to sell the asset but continue using it. What amount should be recognized in profit or loss as a result of the reclassification in 2014? a. 920,000 c. 345,000 b. 460,000 d. 0 Louise Company’s bank statement for the month of April included the following information: Bank service charge for April Check deposited by the entity during April marked “NSI” by the bank and returned Deposits made but not yet recorded by bank Checks written and mailed but not yet recorded by bank

13,000 40,000 132,400 98,700

All deposits in transit and outstanding checks have been properly recorded by the entity. A customer check for P35,000 payable to Louise Company had not yet been deposited and had not been recorded. The bank account balance per ledger P921,300. What amount should be reported as cash on April 30? a. 903,300 c. 834,600 b. 955,000 d. 990,000

A1 PASSERS TRAINING, RESEARCH, REVIEW AND DEVELOPMENT COMPANY// 50

10. On July 1, 2013, Shane Company sold P750,000 of accounts receivable to a finance entity. The finance entity assessed a finance fee of 4% and retained a holdback of 12%. On December 15, 2013, the entity assigned P2,500,000 as a collateral on a P2,000,000 12% annual interest rate bank loan. The bank assessed a 5% service charge on the amount of the loan. On December 31,2013, the allowance for bad debts before adjustment is P32,000 and the balance of accounts receivable, excluding the factored and assigned accounts is P500,000. No assigned accounts have been collected by the end of the year. What total amount was initially received from the financing of accounts receivable? a. 2,530,000 c. 3,530,000 b. 2,505,000 d. 2,620,000 11. Wall Company has significant accounts from three customers, P480,000 due from Step Company, P900,000 due from War Company, and P760,000 due from Hall Company. The entity has other accounts receivable totalling P440,000. The entity determined that the War receivable is impaired by P160,000 and the Hall receivable is impaired by P200,000. The receivable from Step is not impaired. The entity determined that a composite rate of 5% is appropriate to measure impairment on al: other accounts receivable. What is the total impairment of accounts receivable in the current year? a. 382,000 c. 406,000 b. 314,000 d. 360,000 12. On December 31,2013, Bred Company’s ending inventory was P3,000,000 and the allowance for inventory write down before any adjustment was P150,000. Relevant information on December 31, 2013 follow: Product Cost Replacement cost 1 800,000 900,000 2 1,000,000 1,200,000 3 700,000 1,000,000 4 500,000 600,000

Sales price NRV 1,200,000 550,000 1,300,000 1,100,000 1,250,000 950,000 1,000,000 350,000

Normal profit 250,000 150,000 300,000 300,000

What amount of loss on write down is included in cost of goods sold for 2013? a. 50,000 c. 400,000 b. 200,000 d. 250,000 13. The balance in Thunder Company’s accounts payable account on December 31, 2013 was P700,000 before any necessary year-end adjustments relating to the following:  Goods were in transit Thunder from a vendor on December 31, 2013. The invoice cost was P40,000. The goods were shipped “Free Alongside” on December 29, 2013 and were received on January 4, 2014.  Goods shipped “Ex-ship” on December 21,2013 from a vendor to Thunder were received on January 6, 2014. The invoice cost was P25,000.  On December 27, 2013, Thunder wrote and recorded checks to creditors totalling P30,000 that were mailed on January 10,2014. What amount of accounts payable should be reported on December 31, 2013? a. 730,000 c. 765,000 b. 740,000 d. 770,000 14. Bugs Company harvested wool on July 1, 2013. On such date. The fair value less cost of disposal at the point of harvest is P650,000. On December 31, 2013, the wool was still on had and its fair value less cost of disposal and net realizable value were P700,000 and P600,000 respectively. What amount should be recognized as gain on agricultural produce in 2013? a. 150,000 c. 650,000 b. 100,000 d. 600,000 15. On January 1, 2013, Spike Company purchased 1,600 ordinary shares of Lee Company for P528,000. During the year, Lee paid a cash dividend of P13 per share. On December 31, 2-13. Lee shares were selling for P380 per share. Spike purchased the shares and irrevocably designated to classify them at FVTOCI. On October 1, 2014, Lee shares were selling at P400 per share. No dividends were declared by Lee in 2014. What amount of unrealized gain on the investment should be recognized in the 2014 statement of changes in equity? a. 80,000 c. 40,000 b. 56,000 d. 16,000

A1 PASSERS TRAINING, RESEARCH, REVIEW AND DEVELOPMENT COMPANY// 51

16. True Company purchased P1,000,000 of 8% bonds of Blue Company on January 1, 2013, at a discount, paying P922,780. The bonds mature on January 1, 2018, and yield 10% and interest is payable each January 1 and July 1. The entity has a business model whose objective is to hold assets in order to collect contractual cash flows and the contractual terms of the financial asset provide specified dates with regard to cash flows that are solely payments of principal and interest. On December 31, 2013, the market rate of interest is 12%, and the fair value of the bonds is P875,790 at the 12% market rate. What amount should be recognized at interst income for 2013? a. 92,278 c. 73,822 b. 92,585 d. 80,000 17. At the beginning of the year. Space Company purchased a 30% interest in Jam Company for P6,000,000. Jam has 100,000 P100 par value ordinary shares outstanding. On purchase date, the carrying amount of Jam’s net assets was P15,000,000 and that equipment with a 4-year remaining life and inventory were undervalued by P4,000,000 and P2,000,000 respectively. During the year, Jam earned net income of P3, 600,000 and paid dividends of P1,200,000. Only half of the inventory was sold by Jam during the year. At the end of the year, the shares of Jam were trading on an organized exchange for P220 per share. What is the investment income for 2013? a. 1,080,000 c. 360,000 b. 480,000 d. 780,000 18. On January 1, 2013, Raptors Company purchased 25,000 shares of Game Company which represent a 10% interest for P2,000,000. Raptor elected to measure the investment at FVTOCI. Game reported net income of P4,000,000 and paid no dividends in 2013. The fair value of the investment was P1,900,000 on December 31,2013. On January 1, 2014, Raptors paid P5,000,000 for 50,000 additional shares of Game Company. The fair values of the identifiable assets of Game Company equalled carrying amount of P20,000,000 on purchase date expect for land whose fair value exceeded carrying amount by P3,000,000. Game reported net income of P6,000,000 for 2014 and paid dividends of P20 per share on December 31, 2014. What is the carrying amount of the investment on December 31, 2014? a. 7,100,000 c. 7,200,000 b. 6,900,000 d. 5,200,000 19. Fire Company has the Philippine peso is Functional currency. The entity expects to purchased goods from USA for $50,000 on March 31, 2014. Accordingly, the entity is exposed to a foreign currency risk. If the dollar increase before the purchase takes places, the entity will have to pay more pesos to obtain the $50,000 that it will have to pay for the goods. On October 1, 2013, the entity entered into a forward currency contract with a bank speculator to purchase $50,000 in six months for a fixed amount of P2,000,000 or P40 to $1. This forward currency contract is designated as cash flo3w hedge. On December 31, 2013, the exchange rate is P43 to $1. On December 31, 2013, what amount should be reported as derivative asset or liability? a. 100,000 asset c. 100,000 liability b. 150,000 asset d. 150,000 liability 20. At year-end, Candy Company reported cash and cash equivalents comprising cash on hand P500,000 demand deposit P4,000,000, certificate of time deposit P2,000,000, post dated customer’s check P300,000 petty cash fund P50,000, travel’s check P200,000, manager’s check P100,000 and money order P150,000. What total amount of “cash” should be reported at year-end? a. 7,000,000 c. 6,800,000 b. 4,800,000 d. 5,000,000 21. Cane Company acquired a new equipment by exchanging an old equipment: Equipment – old Accumulated depreciation Fair valve of old equipment Cash received on exchange

5,000,000 4,000,000 1,200,000 500,000

What amount should be recorded as cost of the equipment received in exchange? a. 1,200,000 c. 700,000 b. 1,700,000 d. 500,000

A1 PASSERS TRAINING, RESEARCH, REVIEW AND DEVELOPMENT COMPANY// 52

22. Cavs Company fabricated equipment for office use during the current year. The following data were taken from the records: Materials Direct Labor Finished goods 1,000,000 1,500,000 Office equipment 600,000 500,000

Factory overhead amounted to P1,200,000. Normal production of finished goods is 50,000 units. Due to the fabrication of office equipment, finished goods produced totalled 35,000 units only in the current year. The office equipment is to be charged with the overhead which would have been apportioned to the 15,000 units which were not produced. What is the total cost of office equipment? a. 1,100,000 c. 1.460,000 b. 1,400,000 d. 2,300,000 23. Suns Company purchased land for P1,100,000. The entity paid P70,000 to tear down a building on the land. Salvage was sold for P10,500. Legal fees of P6,500 were paid for title investigation and making the purchase. Architect fees were P40,500. Title insurance was P4,500 P12,000. The contractor was paid P1,357,000. A one-time assessment made by the city for sidewalks was P7,500. Suns installed lighting and signage at a cost of P11,000. What is the total cost of the land? a. 1,195,000 c. 1,103,000 b. 1,178,000 d. 1,006,500 24. On January 1, 2013, the city government provided Clippers Company a zero interest P3,000,000 loan with a 3-year term. Interest is payable annually every December 31. The prevailing market rate of interest for this type of loan is 8%. What amount of income from government grant should be recognized in 2013? Round present value factor to 2 decimals, a. 630,000 c. 240,000 b. 189,600 d. 210,000 25. Red Company is constructing a building. Construction began on January 1, 2013 and was completed on December 31, 2013. The average expenditures in 2013 amounted to P3,200,000. The entity borrowed P1,200,000 on January 1, 2013on a 5-year, 12% note to help finance construction of the building. In addition, the entity had outstanding all year a 10%, 3-year, P3,000,000 note payable and a 12%, 4-ygear, P5,000,000 note payable. What amount should be charged to interest expense for 2013? a. 369,000 c. 675,000 b. 900,000 d. 600,000 26. Grey Company purchased a machine on July 1, 2013 for P750,000. The machine had a useful life of 10 years with residual value of P42,000. During 2016, it became apparent that the machine would become uneconomical after December 31, 2020, and that the machine would have no residual. What is the charge for depreciation in 2016? a. 106,200 c. 123,000 b. 114,600 d. 143,250 27. In January 2013, Cement Company purchased a mineral mine for P3,400,000 with removable ore estimated at 2,000,000 tons. The property has an estimated value of P200,000 after the ore has been extracted. The entity incurred P1,000,000 of development cost preparing the mine for production. During 2013, 500,000 tons were removed and 400,000 tons were sold. What is the amount of depletion that should be expensed in 2013? a. 640,000 c. 840,000 b. 800,000 d. 1,120,000 28. Shot Company reported an impairment loss of P1,600,000 in 2012. This loss was related to an item of property, plant and equipment which was acquired on January 1, 2011 with cost of P10,000,000, useful life of 10 years no residual value. On December 31, 2012, the entity reported this asset at P6,400,000 which is the fair value on such date. On December 31, 2013, the entity determined that the fair value of the impaired asset had increased to P7,200,000. The straight line method is used. What amount of gain on reversal of impairment should be reported in 2013? a. 1,600,000 c. 600,000 b. 1,400,000 d. 0 29. On January 1, 2008. Hard Company purchased equipment at a cost of P6,000,000. Depreciation was computed on the straight line basis at 4% per year. On January 1, 2013, the building was revalued at a fair value of P8,000,000. The income tax rate is 30%. What is the revaluation surplus on December 31, 2014? a. 2,128,000 b. 2,240,000 c. 2,880,000 d. 2,016,000 A1 PASSERS TRAINING, RESEARCH, REVIEW AND DEVELOPMENT COMPANY// 53

30. On May 1, 2013, Crimson Company exchanged 20,000 treasury shares of P25 par value for a patent owned by Joker Company. The treasury shares were acquired in 2012 for P450,000. On May 1, 2013, Crimson’s share was quoted at P34, and the patent had a carrying amount of P550,000. What is the initial cost of patent? a. 450,000 b. 500,000 c. 550,000 d. 680,000 31. During the current year, Squadron Company incurred the following costs: Testing in search for process alternative Cost of marketing research for new product Modification of the formulation of a process Research and development services performed by Blue Company for Squadron

32.

33.

34.

35.

36.

350,000 250,000 510,000 425,000

What amount should be reported as research and development expense? a. 1,535,000 c. 1,285,000 b. 1,185,000 d. 860,000 Sick Company acquired an oil rig for P20,000,000 on January 1, 2013. The life of the rig is 10 years and the expected cost to dismantled the rig at the end of 10 years is P4,000,000. The entity is required by law to dismantle the rig after 10 years. The appropriate discount rate is 10%. The PV of 1 at 10% periods is 386. The discount on the liability for dismantling cost should be amortized using the effective interest method. What total amount of expense should be reported for 2013 as a result of these events? a. 2,400,000 c. 2,800,000 b. 2,154,400 d. 2,308,800 Jenkins Company gives its customers coupons redeemable for a poster plus an audio CD. One coupon is issued for each peso of sales. On the surrender of 100 coupons and P50 cash, the poster and CD are given to the customer. It is estimated that 80% of the coupons will be presented for redemption. Sales for 2013 were P7,000,000, and the coupons redeemed totalled 3,400,000. Sales for 2014 were P8,400,000, and the coupons redeemed totalled 8,500,000. Based on past experience, coupons expected to be redeemed in one year will be redeemed in the next year. The entity bought 20,000 posters at P20 each and 20,000 CDs at P60 each. What is the premium liability on December 31, 2014? a. 336,000 c. 126,000 b. 252,000 d. 84,000 Spizike Company had two issues of securities outstanding, namely ordinary shares and 8% convertible bond issue in the face amount of P16,000,000. Interest payment dates of the bond issue are June 30 and December 31. The conversion clause entities the bondholders to receive forty ordinary shares of P20 par value in exchange for each P1,000 bond. On June 30, 2014, the holders of P2,400,000 face value bonds exercised the conversion privilege. The market price of the bond on that date was P1,100 and the market price of the share was P35. The total unamortized bond discount at the date of conversion was P1,000,000 and the equity component when the bonds were issued was P1,200,000. What amount should be credited to share premium-issuance as a result of this conversion? a. 330,000 c. 660,000 b. 510,000 d. 480,000 Faints Company manufactures machinery used in the mining industry. On January 1, 2013, it leased equipment with a cost of P2,000,000 to Gray Company. The 5-years lease calls for a 10% down payment and equal annual payments of P732,600 at the end of each year. The equipment has an expected useful life of 5 years. Gray’s incremental borrowing rate is 10%, and it depreciates similar equipment using the double declining balance method. The cash selling price of the equipment is P3,250,000, and the rate implicit in the lease is 8%, which is known to Gray. What is the lease liability on December 31, 2013? a. 2,777,400 c. 2,426,400 b. 2,192,400 d. 2,484,900 Atlanta Company prepared an aging of accounts receivable on December 31, 2013 and determined that the net realizable value of accounts receivable was P5,000,000. The entity revealed the following information for the current year: Allowance for doubtful accounts - January 1 600,000 Accounts receivable – December 31 6,000,000 Accounts written off 800,000 Recovery of accounts written off 100,000 A1 PASSERS TRAINING, RESEARCH, REVIEW AND DEVELOPMENT COMPANY// 54

What amount should be recognized as doubtful accounts expense for the current year? a. 1,100,000 c. 1,200,000 b. 1,000,000 d. 1,800,000 37. The 12% bonds payable of Motorboat Company had a carrying amount of P8,320,000 on December 31, 2013. The bonds, which had a face value of P8,000,000, were issued at a premium to yield 10%. The entity used the effective interest method of amortization. Interest is paid on June 30 and December 31. On June 30, 2014, the entity retired the bonds at 110 plus accrued interest. What is the loss on retirement in 2014? a. 800,000 c. 544,000 b. 480,000 d. 320,000 38. Rings Company is part of a major industrial group and is known to accurately disclose related party transactions in its financial statements. Remuneration and other payments made to the entity’s chief executive offer during 2013 were: Annual salary Share options and other share-based payments Contributions to retirement benefit plan Reimbursement of travel expenses for business trips

2,000,000 1,000,000 500,000 1,200,000

What total amount should be disclosed as “compensation” to key management personnel? a. 3,500,000 c. 3,000,000 b. 4,700,000 d. 2,500,000 39. On December 31, 2013, a flood at Pamela Company’s only warehouse caused severe damage to its entire inventory. Based on recent history, the entity has a gross profit of 25% of sales. The following information is available from the records for the year ended December 31, 2013: Inventory January 1 Purchases Purchase returns Sales Sales returns Sales allowances

520,000 4,120,000 60,000 5,600,000 400,000 100,000

A physical inventory disclosed usable damaged goods which can be sold for P100,000. Using the gross profit method, what is the estimated cost of goods sold for the year ended December 31, 2013? a. 3360,000 c. 3,900,000 b. 3,830,000 d. 3,825,000 40. Kaye Company leases and operates a retail store. The following information relates to the lease for the year ended December 31, 2013.  The store lease, an operating lease, calls for a base monthly rent of P15,000 on the first day of each month.  Additional rent is computed at 6% of net sales over P3,000,000 up to P6,000,000 and 5% of net sales over P6,000,000 per calendar year.  Net sales for 2013 amounted to P9,000,000.  The entity paid executor costs to the lessor for property taxes of P120,000 and insurance of P50,000 What total amount of the expenses relating to the store lease should be reported for 2013? a. 710,000 b. 680,000 c. 540,000 d. 350,000

A1 PASSERS TRAINING, RESEARCH, REVIEW AND DEVELOPMENT COMPANY// 55



A1 PASSERS TRAINING, RESEARCH, REVIEW & DEVELOPMENT COMPANY 2nd Floor Sommerset Bldg., Lopez Jaena St. Jaro, Iloilo City Tel. No.: (033) 320-2728; 09106547262 Email Address: [email protected] BOARD OF CERTIFIED PUBLIC ACCOUNTANT CERTIFIED PUBLIC ACCOUNTANT Licensure Examination 1. 410,000 2. 175,000 (250,000 X 70%) 3. Depends on the fair value of the financial asset at FVTOCI and amortization of financial asset at amortized cost 4. 22,040,000 5. 3,200,000 (10,000,000 – 6,000,000 – 500,000) 6. Bad debts for the year Bad debts to date – 3rd quarter (6,000,000 x 10%) Bad debts in the 4th quarter 7. Up to extent of P800,000 only 8. Carrying amount 1!1!1!1(5,750,000 – 230,000) Depreciation that would be recognized in 2014 Carrying amount – 12/31/14 as if not held for sale

9. 10. 11.

12.

13. 14. 15. 16. 17.

A A D B D 900,000 (600,000) 300,000B B 4,600,000 (575,000) 4,025,000

FV less cost of disposal – 12/31/14 (4,370,00 – 230,000)

4,140,000

Measurement of equipment – lower amount Carrying amount – 12/31/14 per book Gain on remeasurement 903,300 2,530,000 War Hall Other receivables (480,000 + 440,000 = 920,000 x 5%) Total impairment Required allowance for inventory writedown (3,000,000 – 2,600,000) Allowance before adjustment Loss on inventory writedown 770,000(700,000 + 40,000 + 30,000) 650,000 FV – 12/31/14(800 x 400) HC (528,000 x ½) Cumulative unrealized gain 92,585(46,139 + 46,466) Cost CA of net assets acquired (15,000,000 x 30%) Excess cost Equipment (4,000,000 x 30%) Inventory (2,000,000 x 30%) Excess FV

4,025,000 (3,680,000) 345,000C A A 160,000 200,000 46,000 406,000C

Share in NI (3,600,000 x 30%) Amortization – equipment (1,200,000/4) Amortization – inventory (600,000/2) Excess FV Investment income

1,080,000 (300,000) (300,000) 300,000 780,000D

18. Cost (1,900,000 + 5,000,000) Share in NI (6,000,000 x 30%) Dividends received (20 x 75,000)

400,000 (150,000) 250,000D D C 320,000 (264,000) 56,000B B 6,000,000 (4,500,000) 1,500,000 (1,200,000) (600,000) (300,000)

6,900,000 1,800,000 (1,500,000)

A1 PASSERS TRAINING, RESEARCH, REVIEW AND DEVELOPMENT COMPANY// 56

19. 20. 21. 22. 23. 24. 25.

26. 27. 28.

29.

30. 31. 32.

33. 34.

35.

36. 37. 38. 39. 40.

CA – 12/31/14

7,200,000C

Total shares of Game (25,000/10%) Additional interest (50,000 x 250,000) 100,000 asset (50,000 x2) 5,000,000 (500 + 4,000 + 50 + 200 + 100 + 150) 700,000(1,200,000 – 500,000) 1,460,000 (600,000 + 500,000 + 360,000) 1,178,000 PV of note payable (3,000,000 x 0.9) Income from government grant (2,370,000 x 8%) 3,000,000 x 10% 5,000,000 x 12% 8,000,000

250,000 20% A D C C B 2,370,000 189,000B 300,000 600,000 900,000

Average rate (900,000 / 8,00,000)

11.25%

Specific (1,200,000 x 12%) 144,000 General (2,000,000 x 11.25%) 225000 Capitalizable 369,000 Specific loan interest (1,200,000 x 12%) 144,000 General loan interest (3,000,000 x 10%) + (5,000,000 x 12%) 900,000 Actual interest cost 1,044,000 Capitalized interest (369,000) Amount charged to interest expense 675,000C 114,600 (573,000/5) B 840,000(400,000 x 2.10) C Carrying amount – 21/31/13 as if no impairment (10,000,000 x 7/10) 7,000,000 Carrying amount – 12/31/13 with impairment (6,400,000 – 800,000) 5,600,000 Gain on reversal of impairment loss 1,400,000B FV – 1/1/13 8,000,000 CA – 1/1/13 (6,000,000 – 1,200,000) (4,800,000) Revaluation surplus – 1/1/13 before taxes 3,200,000 DT liability (3,200,000 x 30%) (960,000) Revaluation surplus – 1/1/13 after taxes 2,240,000 Realization in 2013 and 2014 (2,240,000/20 x 2) (224,000) Revaluation surplus – 12/31/14 2,016,000D 680,000(400,000 x 2.10) D 1,285,000 (350,000 + 510,000 + 425,000) C Original cost 20,000,000 PV of dismantling cost (4,000,000 x 0.386) 1,544,000 Total cost of the rig 21,544,000 Depreciation (21,544,000/10) 2,154,400 Interest expense (1,544,000 x 15%) 154,000 Total expense 2,308,800D 126,000(4,200 x 30) C Bonds payable 2,400,000 Share premium – conv. (1,200,000 x 15%) 180,000 Bond discount(1,000,000 x 15%) 150,000 Share capital (2,400 x 40 x 20) 1,920,000 Share - premium – issuance 510,000B Lease liability – 1/1 ‘!3(3,250,000 x 90%) 2,925,000 Less: principal payment Total payment 732,600 Interest (2,925,000 x 8%) 234,000 (498,600) Lease liability – 12/31/13 2,426,000C 1,100,000 A 544,000 C 3,500,000 A 5,200,000 x 75% 3,900,000C 680,000 B A1 PASSERS TRAINING, RESEARCH, REVIEW AND DEVELOPMENT COMPANY// 57



A1 PASSERS TRAINING, RESEARCH, REVIEW & DEVELOPMENT COMPANY 2nd Floor Sommerset Bldg., Lopez Jaena St. Jaro, Iloilo City Tel. No.: (033) 320-2728; 09106547262 Email Address: [email protected] BOARD OF CERTIFIED PUBLIC ACCOUNTANT CERTIFIED PUBLIC ACCOUNTANT Licensure Examination SET B

PRACTICAL ACCOUNTING 2 GENERAL INSTRUCTIONS: 1. This test booklet contains 50 test questions. 2. Read INSTRUCTIONS TO EXAMINEES printed on your answer sheet. 3. Shade only one (1) box for each question on your answer sheets. Two or more boxes shaded will invalidate your answer. 4. AVOID ERASURES. INSTRUCTIONS: 1. Detach one (1) answer sheet from the bottom of your Examinee ID/Answer Sheet Set. 2. Write the subject title “PRACTICAL ACCOUNTING II” on the box provided. 3. Shade Set Box “A” on your answer sheet if your test booklet is Set A; Set Box “B” if your test booklet is Set B. __________________________________________________________________________________________

1. Which of the following statements is incorrect? a. The tax remittance advice is recorded by the agency by crediting subsidy income from National Government b. The gross payroll is advanced to disbursing offer and is recorded by crediting Cash – National Treasury – Modified Disbursement System. c. Upon payment of the office equipment on a account, Due to BIR account shall be credited for the withholding tax d. The unused NCA is to be reverted to the Bureau of Treasury by crediting the Cash – NT – MDS account 2. Faith and hope, a private not-for-profit voluntary health and welfare organization, received the following contributions in 2011: I. P25,000 from donors who stipulated that the money not be spent until 2012. II. P60,000 from donors who stipulated that the contributions be used for the acquisition of equipment, none of which was acquired in 2011. Which of the above events increased temporarily restricted net assets for the year ending December 31, 2011? a. I only c. II only b. Both I and II d. Neither I nor II 3. On January 2, 2011, SD Company signed an agreement to operate as a franchisee of TQ Products ,.Inc,.. for an initial franchisee fee of P937,500 for 7 years. Of this amount, P175,000 was paid when the agreement was signed and the balance payable in four annual payments beginning on December 31, 2011. SD signed a non-interest bearing note for the balance. SD’s rating indicates that he can borrow money at 16% for the loan of this type. Assume that substantial services amounting to P283,500 had already been rendered by TQ Products and that additional direct franchise cost of P25,5000 was also incurred PV factor is 2.80. If the collection of the note is not reasonably assured the net income for the year ended December 31, 2011 is a. P313,435 c. P168,135 b. P228.035 d. P253,535 4. On August 1, 2012, PAUL Corp. Sold a price of land costing P1,350,000 at a gross margin of 66 2/3% above cost. The buyer paid a 20% down payment and ,made four instalments of P90,000 each during the same year. Using the instalment method of accounting, how much is the realized gross profit in 2012? a. 180,000 b. 324,000 c. 486,000 d. 216,000 A1 PASSERS TRAINING, RESEARCH, REVIEW AND DEVELOPMENT COMPANY// 58

5. Tsup – Tsup Corporation filed a bankruptcy petition on January 2009. On March 1, 2009, the trustee provided the following information about the corporation’s financial affairs: Asset Cash Accounts receivable – net Inventories Plant assets – net Total Assets

Book Value P40,000 200,000 300,000 500,000 P1,040,000

Liabilities Liabilities for priority claims Accounts payable – unsecured Notes payable, secured by accounts receivable mortgage payable, secured by all plant assets total Liabilities

Realizable Value P40,000 150,000 140,000 560,000

P160,000 300,000 200,000 440,000 P1,100,000

The amount expected to be available for unsecured claims without priority a. 300,000 c. 140,000 b. 580,000 d. 310,000 6. Krebs Crabs, Inc. Franchisor, entered into a franchise agreement with Liwayway Ligaya, franchisee, on July 1, 2012. The total franchise fees agreed upon is P1,100,000, of which P100,000 is payable upon signing and the balance in four equal annual instalments. It was agreed that the down payment is not refundable, not withstanding lack of substantial performance of services by franchisor. When Kerbs prepares its financial statements on July 31, 2012, the unearned franchise fees to be reported is: a. 0 c. 1,000,000 b. 100,000 d. 1,100,000 7. On December 31,2009 entity A, an SME, acquired 30% of the ordinary shares that carry voting rights of entity Z for P100,000. In acquiring those shares entity A. Incurred transaction of P1,000. Entity A has entered into a contractual arrangement with another party (entity C) that owns 25% of the ordinary shares of entity Z, whereby entities A and C jointly controlled entity Z. Entity A use the cost model to account for its investments in JCE. A fair valuation of the investments in entity Z determined using a reliable earnings multiple approach exists. In January 2010, entity Z declared and paid dividend of P20,000 out of profits earned in 2009. No further dividends were paid in 2010, 2011 and 2012. At December 31, 2010, 2011 and 2012, management assessed the fair values of its investment in entity Z as P102,000, P110,000 and P90,000, respectively. Costs to sell are estimated at P4,000 throughout. Entity A measures its investment in entity Z on December 31, 2011 at? a. P110,000 c. P106,000 b. 101,000 d. P95,000 8. On December 31, 2010, the following figures were taken the trial balances of Ayoshi Cash Receivables Inventory Property and equipment – net Goodwill Current liabilities Long – term liabilities Common stock Additional paid – in capital Retained earning

P160,000 120,000 200,000 400,000 --40,000 140,000 220,000 40,000 440,000

P40,000 120,000 140,000 200,000 60,000 20,000 100,000 200,000 --240,000

On December 312, 2010, Ayoshi issues 10,000 shares of its P10 par value stock for the net assets of Misao. Ayoshi’s stock had a P34 per share fair market value. Ayoshi would also issue bond debentures with face value of P200,000 maturing 3 years from date of issue. Discount related to the bonds issued amounted to P40,000. Ayoshi also paid the following: P50,000 for broker’s fee, P40,000 for pre – acquisition adult fee, P43,000 for legal fees, P36,000 for audit fee for SEC registration of stock issue and P11,000 for printing of stock certificates. Misao hold an equipment that is wroth P80,000 more than its current book value. The retained earnings of Misao on January 1, 2010 amounted to P140,000.

A1 PASSERS TRAINING, RESEARCH, REVIEW AND DEVELOPMENT COMPANY// 59

How much is the total combined assets after the merger? a. P1,500,000 c. P1,280,000 b. P1,240,000 d. P1,320,000 9. On December 3, 2011, Jane Company purchased 70% of the outstanding shares of Jasmine Company for P245,000. On that date, Jasmine Company had P100,00o of capital stock and P250,000 of retained earnings. For 2012, Jane Company had income of P200,000 from its own operations and paid dividends of P100,000. For 2012, Jane Company reported income of P200,000 from its own operations and paid dividends of P100,000. For 2012, Jasmine Company have book value book values approximately equal their book values. The beginning inventory of Jane Company includes P6,000 of merchandise purchased from Jasmine Company on December 31, 2011 at 150% of cost. The ending inventory of Jane Company includes P9,000 of merchandise purchased from Jasmine Company at the same mark up. Jane Company uses FIFO inventory costing. What is the non-controlling interest in Jasmine Company or the year ended December 31, 2012? a. 117,000 c. 107,700 b. 110,700 d. 105,000 10. On January 2, 2011, Davao Corporation purchased 70% of the common stock of Cotabato Company for P4,675,000. At that date, Cotabato had P4,887,500 of common stock outstanding and retained earnings of P1,572,50. Cotabato’s equipment with a remaining life of 5 years had a book value of P2,380,000 and a fair of P2,550,000. Cotabato’s remaining assets had book values equal to their fair values. All intangibles except goodwill are expected to have remaining lives of 10 years. Non-controlling interest shall be measured an fair value. The income and dividend figures for both Davao and Cotabato are as follows: Income Dividend Davao: 2011 P1,572,500 P425,000 2012 1,785,000 510,000 Cotabato: 2011 340,000 85,000 2012 569,500 127,5000 Davao’s income as shown does not include any dividend income from Cotabato. Davao’s retained earnings balance at the date of acquisition was P5,858,500. On December 31, 2012, the consolidated retained earnings is: a. P8,821,300 c. P8,993,850 b. P8,970,050 d. P29,017,650 11. Fish Ball Co. Charges P90,000 for a franchise with P18,000 paid when the agreement is signed and the balance in four annual payments. The present value of the annual payments, discounted at 9% is P58,315. The franchises has the right to purchase P20,000 of equipment for P16,000. If collectability of the payments is reasonably assured and substantial performance by Fish Ball has occurred, what is the amount of revenue from franchise fee that should be recognized? a. 81,563 c. 78,315 b. 76,315 d. 72,000 12. PAX has the following foreign currency transaction during 2012: Merchandise was purchased from a foreign supplier on Jan. 20, 2012 for the peso equivalent of P900,000. The invoice was paid on March 31, 2012, at the equivalent of P960,000. On July 1, 2012, PAX borrowed the equivalent of P5,000,000 equivalent by a note that is payable in the lender’s local Currency on July 1, 2012. On Dec. 31, 2012, the equivalent of the principal amount and accrued interest were P5,200,000 and P260,000 respectively. Interest on the notes is 10% per annum. In the 2012 income statement of PAX what amount should be reported as foreign exchange loss? a. 0 c. 270,000 b. 210,000 d. 80,000 13. ON Jan. 5, 2012, P sold to its 80% owned subsidiary, S Corp. , a machine for P120,000. At that time the machine had a net book value of P90,000. S estimated the remaining life of the machine to be six years. Assume that in 2012, P and S reported a profit of P80,000 and P100,000 respetively. Determine the consolidated net income for 2013? a. 119,000 b. 155,000

c. 144,000 d. 135,000

A1 PASSERS TRAINING, RESEARCH, REVIEW AND DEVELOPMENT COMPANY// 60

14. P owns 80% interest in ABC Corp. In the year 2010, the two firms reported profit own operations as follows: P 90,000 ABC 75,000 Record show that P shipped merchandise to ABC billed for P360,000 the cost is P300,000 and 20% of these goods are not yet sold as of Dec. 31, 2012 ABC constructed a warehouse for P at a cost P120,000 and billed the latter for P150,000 on January 1, 2012. The asset has an estimated useful life of 5 years. How much is the consolidated net income at the end of 2012? a. 129,000 c. 141,000 b. 165,000 d. 118,800 15. D and EW entered into partnership on February 1, 2012 by the investing the following: Diaz D 15,000

Cash Inventory Land Building Furniture and Fixture

Esteban B 45,000 15,000 65,000

100,000

The agreement between D and E provides that profits and losses are to on divided into 40% and 60% to D and E, respectively. The partnership is to assume the P30,000 mortgage loan an the building. Assuming that E invests P50,000 cash and each partner is to be credited for the full amount of the net assets invested the total capital of the partnership is: a. 210,000 c. 260,000 b. 250,000 d. 290,000 16. A,B and C decided to dissolve the partnership on Nov. 30, 2012. Their capital balances and profit ratio on this date follow:

A B C

Capital P50,000 60,000 20,000

D&E ratio 40% 30% 30%

The Net income from Jan to Nov. 30, 2012 is P44,000. Also on this date, cash end liabilities are P40,000 and P90,000 respectively. For A to receive P55,200 in full settlement of this interest in the firm, how much must be realized from the sale of the firms non cash assets? a. 177,000 c. 193,000 b. 187,000 d. 196,000 17. Harris Company has the following information for July: Units started Beginning Work in process:(35% complete) Normal spoilage (discrete) Abnormal spoilage Ending Work in process:(70% complete) Transferred out Beginning Work in process Costs: Material Conversion

100,000 20,000 3,500 5,000 14,500 97,000

units units units units units units

P15,000 10,000

All material are added at the start of the production process. Harris Company insects goods at 75 percent completions to conversation.

A1 PASSERS TRAINING, RESEARCH, REVIEW AND DEVELOPMENT COMPANY// 61

Assume that the costs per EU for material and conversion are P1.00 and P1.50, respectively. Using FIFO, what is the total cost assigned to the transferred-out units (rounded to the nearest dollar)? a. P245,750 c. P237,000 b. P244,438 d. P224,938 18. Palos Company opened its Cabu branch a year ago. At year end, the branch summarized operating data as follows: Sales, P364,000. Shipments from home office, P148,500; Purchases, P137,500; Expenses, P71,500; Ending inventory, P82,500 (of which P16,500 from outsiders); beginning inventory, P60,000 (of which P45,000 came from office.) The overstatement in branch cost of sales assuming shipments by home office are made at 20% Gross Profit rate is: a. P25,500 c. P16,250 b. P21,000 d. P21,250 19. The true net income of the branch is: a. P45,250 c. P50,250 b. P50,000 d. P54,500 20. Nica provide the following information for the transaction occurred during August. The production plant uses the JIT costing system. - Raw materials costing P750,000 were purchased - All direct materials costing P750,000 were requisitioned for production. - Direct labor costs of P500,000 were incurred. - Actual factory overhead costs amounted to P2,487,500. - Applied conversion cost totalled P3,250,000. This includes the direct labor cost. - All units are completed and immediately sold. The total RIP used to be back flushed to FG and the adjusted COGS, respectively a. P750,000; 3,737,500 c. 4,000,000; 3,737,500 b. 4,000,000; 4,262,500 d. 750,000; 4,262,500 21. A Corporation received a promissory note denominated in foreign currency from the sales made to a Singaporean customer. The following were the transactions: (In Singapore Dollars). On December 1, A Corporation sold merchandise to a Singaporean customer for 60-day, 12% promissory note for $32,000, at a buying rate of $1 to P31.20. on December 31, the buying spot rate is $1 to P34.85. on January 30 the buying spot rate is P33.75. on the settlement date how much is the forex gain or loss? a. P35,552 gain c. P35,904 gain b. P35,904 loss d. P35,552 loss 22. On November 1, 2009, Galaxy Philippines took delivery from a Thailand from of inventory costing 225,000 baht. Payment is due on January 30, 2010. Concurrently, Galaxy Philippines paid P2,025 cash to acquire a 90-day call option for 225,000 Thailand baht.

Spot rate (market price) Strike price (exercise price) Fair value of call option

11/1/2009 P1.20 1.20 P2,025

12/31/2009 P1.22 1.20 P4,950

1/30/2010 P1.23 1.20 P6,750

What is the intrinsic value and time value of option on December 31, 2009?

A. B.

Intrinsic value Time Value P4,500 P450 4,950 0

Intrinsic Value C. P450 D. 0

Time Value P4,500 4,950

23. SSS Company produces two products (A and B). Direct material and labor costs for Product A total P35 (35 (which reflects 4 direct labor hours); direct material and labor costs for product B total P22 (which reflects 1.5 direct labor hours). Three overhand functions are needed for each product... product A uses 2 hours of function 1 at P10 per hour, 1 hour of function 2 at P7 per hour, and 6 hours of Function 3 at P18 per hour. Product B user 1, 8, and 1 hours of function 1, 2, 3, respectively. SSS produces 800 units of A and 8,000 units of B each period. If total overhead is assigned to A and B on the basis of units produced, Product A will have an overhead cost per unit of a. P88.64. b. P123.64. c. P135.00. d. None of the responses are correct. A1 PASSERS TRAINING, RESEARCH, REVIEW AND DEVELOPMENT COMPANY// 62

24. The following information is available from the Fitzgerald Company: Actual OH Fixed OH expenses, actual Fixed OH expenses, budgeted Actual hours Standard hours Variable OH rate per DLH

P15,000 P7,200 P7,000 3,500 3,800 P2.50

Assuming that Fitzgerald uses a three-away analysis of overhead variance, what is the overhead spending variance? a. P750 F c. P950 F b. P750 U d. P1,500 U 25. The following March Information is available for Scott Manufacturing Company when it produced 2,100 units: Standard: Material Labor

2 pounds per unit @ P5.80 per pound 3 direct labor hours per unit @ P10.00 per hour

Actual Material Labor

4,250 pounds purchased and used @ P5.65 per pound 6,300 direct labor hours at P9.75 per hour

What is the material price variance? a. P637.50 U c. P630.00 U b. P637.50 F d. P630.00 F 26. Light of Jesus operates a branch in Cagayan de Oro City. Selected accounts taken from Dec. 31, 2010 financial statements of light of Jesus and its branch follows:

Sale Shipments to Branch Shipments from home office Inventory, Jan, 1 Inventory, Dec. 31 Purchases Allowance for overvaluation before Adjustment Expenses

Home Office P6,900,000 4,750,000 800,000 640,000 6,800,000 452,500 356,000

Branch P3,765,000 2,187,500 120,000 250,000 1,000,000

250,000

The ending inventory of the branch includes P120,000 purchased from outside suppliers. The consolidated net income is: a. P1,791,500 b. P2,220,000 c. P2,218,00 d. P2,244,000 27. On December 31, 2005 a foreign subsidiary in Hongkong submitted the following balance sheet stated in foreign currency: Total assets Total Liabilities Common stock Retained Earnings

$500,000 100,000 250,000 150,000

The exchange rate are: Current rate Historical rate Weight average

P3.40 3.10 3.00

A1 PASSERS TRAINING, RESEARCH, REVIEW AND DEVELOPMENT COMPANY// 63

Assuming the functional currency of the subsidiary is the not the currency of the hyperinflationary economy was used and the retained earnings of the subsidiary on December 31, 2005 translated to Peso is P460,000. What amount of Cumulative translation adjustment is to be reported in the consolidated balance sheet on December 31, 2005? a. 25,000 c. 50,000 b. 10,000 d. 125,000 28. O company sold merchandise for 90,000 pounds from a vendor in London on November 1, 2009. Payment in British pounds was due on January 30, 2010. On the same date, O entered into a 90 day futures contract to sell 90,000 pounds from a bank. Exchange rate for pound on different dates are as follows: Nov.1 Dec.31 Jan.31 Spot rate P71.4 P72.7 P71.9 30 day futures 72.3 72.5 73.2 60 day futures 71.8 72.2 72.6 90 day futures 70.6 72.6 73.4 How much is the net forex gain or loss on January 31, 2010? a. P18,000 loss c. P9,000 loss b. P18,000 gain d. P9,000 gain 29. PPP Company produces 50,000 units of Product Q and 6,000 units of Product Z during a period. In that period, four set-ups were required for color changes. all units of Product Q are black, which is the color in the process at the beginning of the period. A set-up was made blue units of Product Z; a set-up was made for 4,500 red units of Product Z; a set-up was made for 500 green units of Product Z. A set-up was then made to return the process to its standard black coloration and the units of Product Q were run. Each setup costs P500. If set-up cost is assigned on a volume basis for the department, what is the approximate per unit set-up cost for Product Z? a. P.010 c. P.040 b. P.036 d. None of the responses are correct 30. Uniform Company has developed standard overhead costs based on a capacity of 180,000 machine hours as follows: Standard costs per unit: Variable portion Fixed portion

2 hours @ P3 = 2 hours @ P5 =

P6 10 P16 During April, 85,000 units were scheduled for production, but only 80,000 units were actually produced. The following data relate to April: Actual machine hours used were 165,000 Actual overhead incurred totalled P1,378,000(P518,000 variable plus P860,000 fixed). All inventories are carried at standard cost. The fixed overhead spending variance for April was: a. P40,000 U c. P60,000 F b. P40,000 F d. P60,000 U 31. Forrest Company uses a standard cost system for its production process and applies overhead based on direct labor hours. The following information is available for August when. Forrest made 4,500 units: Standard: DLMH per unit Variable overhead per DLH Fixed overhead per DLH Budgeted variable overhead Budgeted fixed overhead

2.50 P1.75 P3.10 P21,875 P38,750

Actual: Direct labor hours 10,000 Variable overhead P26,250 Fixed overhead P38,000 Using the two-variance approach, what is the controllable variance? a. P5,812.50 U b. P5,812.50 F c. P4,375.00 U d. P4,375.00 F A1 PASSERS TRAINING, RESEARCH, REVIEW AND DEVELOPMENT COMPANY// 64

32. Ratcliff Company produces two products from a joint process: X and Z. Joint processing costs for this production cycle are P8,000. Disposal Sales price Cost per Further Final sale per yard at yard at processing price per Yards split-off split-off per yard yard X 1,500 P6.00 P3.50 P1.00 P7.50 Z 2,200 5.00 3.00 3.00 11.25 If X and Z are processed further, no disposal costs will be incurred or such costs will be borne by the buyer. Using approximated net realizable value at split-off, what amount of joint processing cost is allocated to X (round to the nearest dollar)? a. P3,090 b. P5,204 c. P4,000 d. P2,890 33. Whale Company manufactures products X and Y from a joint process that also rears a by product, Z. Revenue from sales of Z is treated as a reduction of joint costs. Additional information is as follow: Products X Y Z Total Units produced 20,000 20,000 10,000 50,000 Joint costs ? ? ? P262,000 Sales value at Slit-off P300,000 P150,000 P10,000 P460,000 Joint costs were allocated using the sales value at split-off approach. The joint costs allocated to product X were a. P84,000 b. P100,000 c. P150,000 d. P168,000 34. The following information is available for Hazel Company for April: Started this month 80,000 units Beginning WI (40% complete) 7,500 units Normal spoilage (discrete) 1,100 units Ending WI (70% complete) 900 units Beginning Work in Process Costs: Material P10,000units Conversion 13,800 units Current Costs: Material P120,000units Conversion 350,000units All material are added at the start of production and the inspection point is at the end of the process. What is cost assigned to ending unit for material using weighted average? a. P1.49 c. P1.56 b. P1.63 d. P1.44 35. What is the cost assigned to ending inventory using FIFO? a. P75,920 c. P56,42C b. P58,994 d. P53,144 36. The following information is available for Mathis Company for the current year: Beginning Work in process Costs of Beginning Work in process: (75% complete) 14,500 units Material P25,100 Started 75,000 units Conversion 50,000 Ending Work in process current Costs: (60% complete) 16,000 units Material P120,000 Abnormal spoilage 2,500 units Conversion 300,000 (continuous) Transferred out 66,000 units A1 PASSERS TRAINING, RESEARCH, REVIEW AND DEVELOPMENT COMPANY// 65

All materials are added at the start of production. What is the cost per equivalent unit for material using weighted average? a. P1.72 c. P1.77 b. P1.62 d. P2.07 37. V and A are partners having capital balances of P150,000 and P180,000, respectively, and sharing profits and losses equally. They admit 1. to a 1/3 interest in the partnership capital and profits for an investment of P195,000. If the asset revaluation method is used in recording the admission of the partnership. a. L capital will be P175,000 c. A capital will be P210,000 b. Total capital will P525,000 d. Asset revaluation will be at P45,000 38. A,B and C are partners with average capital balances during 2012. P472,500 ; P238,650 ; and P162,350, respectively. The partners receive 10 interest on their average capital balances ; after deducting salaries of P122,325 to A and P82,625 to C, the residual profit or loss is divided equally. In 2012, the partnership had net loss of P125,624 before the interest and salaries to partners. By what amount should A and C capital account change?

A. B. C. D.

A, capital

B, capital

40,844 decrease 28,358 decrease 29,476 increase 30,267 increase

31,237 decrease 32,458 increase 17,536 increase 40,448 decrease

39. In 2011, Dreambuilders, Inc. was contracted to build the private road network of ALBABANG Subdivision for P350 million. The product was expected to be finished in 2 years and the contract provide for:  A 5% mobilization fee (to be deducted from the last billing), payable within 15 days from the contract signing.  A retention provision of 10% on all billings, payable with the final bill after the completed contract is accepted .  Payment of progress billings within 30 days from acceptance. Dreambuilders, Inc. estimated a 25% gross margin on the project. By the end of the year, Dreambuilders had presented progress billings corresponding to 50% completion. ALABANG accepted all the bills presented, except one for 10% which was accepted on January 7 of the next year. With the exception of the second to the last billing for 10% which was due January 13 of the next year, all accepted billings were settled. As of December 31, 2011, Dreambuilders, Inc. construction in progress, net of billing, from the project under the percentage of completion method amounted to: a. P35 million b. P140 million c. P0 d. P175 million 40. on January 2, 2010, Panaad Company acquired 80% interest in Sarabia Company for P4,125,000 cash. On the date the outstanding; capital stock and retained earnings of Panaad Company and Sarabia Company are as follows: Panaad Sarabia Common shares P2,250,000 P1,312,000 Share premium 1,500,000 Retained earning 525,000 3,187,500 There was no issuance of capital stock during the year. Non-controlling interest is initially measured at fair value. Fair values of the following assets of Sarabia expanded their book values as follows: Inventories, P210,000; Property and equipment (useful life 10 years). P127,500. All other assets and liabilities are fairly valued. Goodwill if any is not impaired. On December 31, 2010 the two companies reported the following operating results: Panaad Sarabia Net Income P1,785,000 P975,000 Dividends paid 525,000 262,500 What is the consolidated stockholders’ equity to be reported in the consolidated statement of financial position on December 31, 2010? a. P10,651,800 b. P13,500,000 c. P7,035,000’ A1 PASSERS TRAINING, RESEARCH, REVIEW AND DEVELOPMENT COMPANY// 66

d. P11,781,000 41. Tropical Company manufacturing three products a joint process which costs P25,000. Each product can be sold at slit-off or processed further and then sold. 10,000 units of each product are manufacturing. The following information is available for the three products. Sales Value Separable Sales Value Products at Split-off Processing at Completion Costs after Split-off A P12 P9 P21 B 10 4 17 C 15 6 19 To maximize profits, which products should Tropical process further? a. Product A only b. Product B only c. Product C only d. Product A,B, and C 42. Which of the following statements is incorrect? a. In the construction of building by contract, Advances to Contractor account may be debited when billings are received. b. In the construction of building by contract, Construction in Progress account it credited to recognize the building account. c. Issued purchase order for the office equipment requires only a memo entry in RAOCO. d. The agency enters the obligation for bank charges in RAOFE; Cash in Bank- LCCA account is credited in the regular agency books. 43. A non profit organization had the following cash contributions and expenditures in 2012: Unrestricted cash contributions P300,000 Restricted cash contributions for the acquisition of property 200,000 Cash expenditures to acquire property 200,000 The statement of cash follows should include which of the following amounts? a. Operating P700,000, investing (P200,000), and Financing P0. b. Operating P500,000, investing P0, and financing P0. c. Operating P500,000, investing (P200,000) d. Operating P0, Investing P500,000 and Financing P200,000 44. On July 1, 2012, XYZ Construction Corp. Contracted to build in office building for ABC, Inc. for total contract price of P975,000. 2012 2013 2014 Contract cost incurred to date P75,000 2,600,000 P1,050,000 Estimated costs to complete the contract 675,000 400,000 Billings to ABC, Inc. 150,000 550,000 275,000 How much is the Construction in Progress account balance at December 31, 2013, using the percentage of completion method? a. P900,000 b. P575,000 c.P825,000 d. P350,000 45. MOBY Company which began operation on Jan. 1, 2012, appropriately uses the instalment sales method of accounting. The following data are available for 2012. Instalment Accounts Receivable 12/31/12 DGP, 12/31/12 (before recognition of RGP) GPR on sale The cash collection during the year amounted to: a. a. 300,000 c. 600,000 b. b. 450,000 d. 432,000 46. Diane and Pinky join for the sale of certain mer5chandise. The participants agree to the following: a. Diane shall be allowed a commission of 10% on his net purchase. b. The participants shall be allowed commissions of 2.5% on their respective sales. c. Diane and Pinky shall divide the profit or loss 60% and 40%, respectively Joint Venture Transaction follows: Dec. 1 Diane makes cash purchase of P57,000 3 Pinky pays venture expenses of P9,000 5 Sales are as follows: Diane- P48,000; Pinky-P36,000. The participants keep their own cash receipts 6 Diane returns unsold merchandise and receives P15,000 cash 15 The participants make cash settlement A1 PASSERS TRAINING, RESEARCH, REVIEW AND DEVELOPMENT COMPANY// 67

In the final settlement, what amount would Pinky pay Diane? a. 14,100 b. 14,880 c. 14,890 d. 15,100 47. On March 2011 entities SME A and SME B each required 30 percent of the ordinary shares that carry voting rights at a general meeting of shareholders of entity Z of P300,000. Entities A and B immediately agreed to share control over entity Z. On 31 December 2011 entity Z declared a dividend of P100,000 for the year 2011. Entity Z reported a profit of P80,000 for the year ended 31 December 2011. At 31 December 2011 the recoverable amount of each venture’s investment in entity Z is P290,000 (calculation: fair value P293,000 less cots to sell P3,000). There is no published price quotation for entity Z. Assuming that entity Z earned its profit evenly through the year, under Cost Method, How much will be recognized in profit (loss) by each venture as a result of the investment? a. a. 30,000 c. 20,000 b. b. 16,667 d. 38,333 48. On January 1, 2011, A acquired a 50% interest in B for P60 million. A already held a 20% interest which had been acquired for P20 million but which was valued at P24 million at January 1, 2011. The fair value of the NCI at January 1, 2011 was P30million, and the fair value of the identified net assets of B was P100 million. How much is the goodwill to be recognized as a result of the business combination? A. A. 3,000,000 B. 7,000,000 C.0 D.4,000,000 49. Kobe Company acquired the net assets of Lakers Corporation on January 1, 2011. Since the parties cannot agree on the definite value of the company in terms of potential future earnings, they agreed to include in the purchase agreement a provision for contingent consideration. Whereby the acquirer will apply an additional cash payments on January 1, 2013 equal to twice the amount by which average earnings of Lakers exceed P250,000 per year, prior to January 1, 2013. Net income was P500,000 in 2011 and P600,000 in 2012. Assume that the liabilities recorded on January 1, 2011, include an estimated contingent liability amounting to P400,000. What was the entry made by Kobe in January 1, 2013? A. Liability form contingent consideration 400,000 Cash B. Goodwill 200,000 Liability from contingent consideration 400,000 Cash C. Liability from contingent consideration 400,000 Loss on contingent consideration 200,000 Cash D. Goodwill 600,000 Cash

400,000

600,000

600,000 600,000

50. Pete Enterprises owns 60% of the outstanding stock of Susie Company, which it purchased for P50,000 above the underlying book value P720,000 on December 31, 2008. For the year 2011, Susie included in its net income P90,000 of unrealized gain on a year-end sale of depreciable assets to Pete. The NCI of Susie was assigned P12,000 of income in the 2011 consolidated financial statements. The excess allocated to equipment is amortized over 20 years. What is the net income reported by Susie for 2011? a. 125,000 b. 120,000 c. 155,000 d. 150,000

A1 PASSERS TRAINING, RESEARCH, REVIEW AND DEVELOPMENT COMPANY// 68

A1 PASSERS TRAINING, RESEARCH, REVIEW AND DEVELOPMENT COMPANY// 69

View more...

Comments

Copyright ©2017 KUPDF Inc.
SUPPORT KUPDF